Exam 3 material, Old's Maternity Ch 6

Pataasin ang iyong marka sa homework at exams ngayon gamit ang Quizwiz!

The nurse is instructing parents of a newborn about voiding and stool characteristics. Which of the following would be considered an abnormal pattern? 1. Large amounts of uric acid crystals in the first days of life 2. At least 6 to 10 wet diapers a day after the first few days of life 3. 1 to 2 stools a day for formula-fed baby 4. Urine that is straw to amber color without foul smell

Answer: 1 Explanation: 1. Small, not large, amounts of uric acid crystals are normal in the first days of life.

The nurse is teaching a group of young women how to prevent urinary tract infections. What will the nurse include in the teaching? Note: Credit will be given only if all correct and no incorrect choices are selected. Select all that apply. 1. The importance of good hygiene 2. How to recognize the signs and symptoms 3. How to take prescribed antibiotics 4. Fluids are restricted to 1000 ml 5. To urinate only when the urge is strong

Answer: 1, 2, 3 Explanation: 1. The nurse should make sure that women are aware of good hygiene practices, and should provide information on other ways to avoid UTI. 2. Women need to know the signs and symptoms of UTIs to secure prompt treatment. 3. If an infection is present, the woman needs information on how to take antibiotics, and needs to know to complete all of them.

Extended use combined oral contraceptives (COCs) reduce the side effects of COCs such as which of the following? 1. Cramping 2. Hypertension 3. Breast tenderness 4. Bloating

Answer: 2 Explanation: 2. Complications of COCs include: myocardial infarction, stroke, blood clots, and hypertension.

5) What is the primary carbohydrate in mammalian milk that plays a crucial role in the nourishment of the newborn? 1. Colostrum 2. Lactose 3. Lactoferrin 4. Secretory IgA

Answer: 2 Explanation: 2. Lactose is the primary carbohydrate in mammalian milk.

A nurse is evaluating the diet plan of a breastfeeding mother. Which beverage is most likely to cause intolerance in the infant? 1. Orange juice 2. Milk 3. Decaffeinated tea 4. Water

Answer: 2 Explanation: 2. Often fussy breastfeeding or cow's milk-based formula-fed infants are switched to a lactose-free formula because of concerns about lactose intolerance.

Under the influence of progesterone, which of the following occur? Note: Credit will be given only if all correct and no incorrect choices are selected. Select all that apply. 1. Temperature decreases 2. Cervix secretes thick, viscous mucus 3. Breasts prepare for lactation 4. Breast glandular tissue decreases in size 5. Vaginal epithelium proliferates

Answer: 2, 3, 5 Explanation: 2. Under the influence of progesterone, the cervix secretes thick, viscous mucus. 3. Under the influence of progesterone, breasts prepare for lactation. 5. Under the influence of progesterone, vaginal epithelium proliferates.

The nurse is educating a group of female adolescents regarding sexually transmitted infections. The nurse knows that learning was achieved when an individual states that the most common symptom is which of the following? 1. Menstrual cramps 2. Heavy menstrual periods 3. Flu-like symptoms 4. Usually there are no signs or symptoms

Answer: 4 Explanation: 4. It is common for women to experience no signs or symptoms when they have contracted a sexually transmitted disease.

The OB-GYN nurse is teaching a client at the clinic that use of a spermicide has an increased effectiveness if used with which other items? Note: Credit will be given only if all correct and no incorrect choices are selected. Select all that apply. 1. A non-water-based lubricant 2. A diaphragm 3. A contraceptive sponge 4. Prophylactic antibiotics 5. A condom

2, 3, 5 Explanation: 2. Barrier contraceptives such as a diaphragm act by blocking the transport of sperm and are often used in conjunction with a spermicide. 3. Barrier contraceptives such as the contraceptive sponge act by blocking the transport of sperm and are often used in conjunction with a spermicide. 5. Barrier contraceptives such as the condom act by blocking the transport of sperm and are often used in conjunction with a spermicide.

The clinic nurse teaches the pregnant client being treated for trichomoniasis about the risks to her pregnancy due to this infection. Which statement would indicate successful teaching? 1. "I am at risk of having a preterm birth because of this infection." 2. "I might need to have my membranes ruptured because of this infection." 3. "I am at risk of having a baby with a high birth weight." 4. "I may have intercourse with my husband while bring treated for this infection."

Answer: 1 Explanation: 1. There is a risk of preterm birth and rupture of membranes.

The nurse should inform the parents of a newborn that they should call their healthcare provider when which of the following occurs? Note: Credit will be given only if all correct choices and no incorrect choices are selected. Select all that apply. 1. Continual rise in temperature 2. Decreased frequency of stools 3. Absence of breathing longer than 20 seconds 4. Lethargy 5. Refusal of two feedings in a row

Answer: 1, 3, 4, 5 Explanation: 1. Parents should call their healthcare provider due to a continual rise in temperature. 3. Parents should call their healthcare provider in the absence of breathing longer than 20 seconds. 4. Parents should call their healthcare provider if the newborn exhibits lethargy and listlessness. 5. Parents should call their healthcare provider if the newborn has refused of two feedings in a row.

A nurse is teaching a class on the different types of uterine bleeding. The nurse explains that which of the following is one of the causes of abnormal uterine bleeding? 1. Iron-deficiency anemia 2. Polyps 3. Heavy periods every 2 months 4. Spotting between periods

Answer: 2 Explanation: 2. A classification system has been developed for the causes of AUB using the acronym PALM-COEIN. The P stands for Polyps.

The nurse is caring for a client hospitalized for pelvic inflammatory disease. Which nursing intervention would have priority? 1. Encourage oral fluids 2. Administer cefotetan IV 3. Enforce bed rest 4. Remove IUC, if present

Answer: 2 Explanation: 2. Administration of medications to treat the disease is the first priority.

The nurse caring for a postterm newborn would not perform what intervention? 1. Providing warmth 2. Frequently monitoring blood glucose 3. Observing respiratory status 4. Restricting breastfeeding

Answer: 4 Explanation: 4. Breastfeeding is an appropriate means of feeding for the postterm newborn.

Which of the following is a sign of dehydration in the newborn? 1. Slow, weak pulse 2. Soft, loose stools 3. Light colored, concentrated urine 4. Depressed fontanelles

Answer: 4 Explanation: 4. Depressed fontanelles are a sign of dehydration in the newborn.

The nurse is preparing a handout for female adolescents on the menstrual cycle. What phase of the cycle occurs if fertilization does not take place? 1. Menstrual 2. Proliferative 3. Secretory 4. Ischemic

Answer: 4 Explanation: 4. The ischemic phase occurs if fertilization does not occur.

The nurse is working with an adolescent parent. The adolescent tells the nurse, "I'm really scared that I won't take care of my baby correctly. My mother says I'll probably hurt the baby because I'm too young to be a mother." What is the best response by the nurse? 1. "You are very young, and parenting will be a challenge for you." 2. "Your mother was probably right. Be very careful with your baby." 3. "Mothers have instincts that kick in when they get their babies home." 4. "We can give the baby's bath together. I'll help you learn how to do it."

Answer: 4 Explanation: 4. This response is best because bathing the newborn offers an excellent opportunity for teaching and welcoming parent involvement in the care of their baby.

Which questions are appropriate for the nurse to ask during a cultural assessment of a client who is new to the clinic? Note: Credit will be given only if all correct and no incorrect choices are selected. Select all that apply. 1. What genetic and other biological differences affect caregiving? 2. Which family member must be consulted for decisions about care? 3. What type of health provider is the most appropriate? 4. Does the client have beliefs or traditions that might impact the care plan? 5. Are communications patterns established?

Explanation: 2. It is important the nurse recognize cultural differences in regard to which family member must be consulted for decisions about care. 3. Some cultures do not allow a person of the opposite gender to touch the client. Cultural sensitivity will recognize and allow for this. 4. The nurse must be aware of traditions and beliefs that might impact the care plan.

A client asks her nurse, "Is it okay for me to take a tub bath during the heavy part of my menstruation?" What is the nurse's correct response? 1. "Tub baths are contraindicated during menstruation." 2. "You should shower and douche daily instead." 3. "Either a bath or a shower is fine at that time." 4. "You should bathe and use a feminine deodorant spray during menstruation."

Explanation: 3. Bathing, whether it is a tub bath or a shower, is as important (if not more so) during menses as at any other time.

The nurse has received the shift change report on infants born within the previous 4 hours. Which newborn should the nurse see first? 1. 37-week male, respiratory rate 45 2. 8 pound 1 ounce female, pulse 150 3. Term male, nasal flaring 4. 4-hour-old female who has not voided

Explanation: 3. Nasal flaring is an indication of respiratory distress. The nurse must be immediately available to provide appropriate interventions for a newborn in distress.

What interventions would the nurse apply to support the breastfeeding mother? Note: Credit will be given only if all correct choices and no incorrect choices are selected. Select all that apply. 1. Assist the mother to begin breastfeeding within the first hour after birth. 2. Have the baby returned to the nursery after feeding so that the mother can get adequate rest. 3. Teach the mother to recognize and respond to early infant feeding cues. 4. Inform the mother about community resources that support breastfeeding. 5. Instruct the mother to avoid eating foods that might upset the newborn's stomach

1, 3, 4 Explanation: 1. Throughout the first 2 hours after birth, but especially during the first hour of life, most infants are usually alert and ready to breastfeed. 3. The new mother should be taught to recognize and respond to early infant feeding cues. The timing of newborn feedings is ideally determined by physiologic and behavioral cues rather than a set schedule. 4. It is important that parents receive verbal and written instructions and community resource information to which they can later refer.

A newborn delivered at term is being discharged. The parents ask the nurse how to keep their baby warm. The nurse knows additional teaching is necessary if a parent states which of the following? 1. "A quick cool bath will help wake up my son for feedings." 2. "I can check my son's temperature under his arm." 3. "My baby should be dressed warmly, with a hat." 4. "Cuddling my son will help to keep him warm."

Answer: 1 Explanation: 1. Cool baths will chill a newborn, and should not be given. Bathing under warm water is ideal.

The nurse is instructing a young client on avoiding toxic shock syndrome. Education was successful when the client makes which statements? Note: Credit will be given only if all correct and no incorrect choices are selected. Select all that apply. 1. "I will wash my hands before inserting a tampon." 2. "I will change my tampon every 3-6 hours." 3. "I will not touch the part of the tampon I insert." 4. "I will just put the used tampon in the trash." 5. "I will take prophylactic antibiotics if needed."

: 1, 2, 3 Explanation: 1. Washing hands before inserting or removing a tampon is correct. 2. Changing the tampon every 3-6 hours will help prevent toxic shock syndrome from developing. 3. After the tampon is unwrapped, the client should avoid touching the portion of the tampon to be inserted into the vagina.

A couple who came to the United States two years ago with their two children are seeing the nurse in the community clinic. The nurse knows their family is acculturating when the mother makes which statement? 1. "The children are much less well-behaved than they used to be." 2. "Our diet now includes hamburgers and French fries." 3. "We celebrate the same holidays that we used to at home." 4. "When the children leave the house, I worry about them."

: 2 Explanation: 2. Inclusion of fast food in the diet is an indication of acculturation, because it shows a belief in the nutritional value of these foods and an acceptance of purchasing fast food as equivalent in value to home-cooked meals.

Which client in the gynecology clinic should the nurse see first? 1. 22-year-old with fever, hypotensive, using tampons 2. 15-year-old, no menses for past 4 months 3. 18-year-old seeking information on contraception methods 4. 31-year-old, reports increasing dyspareunia

Answer: 1 Explanation: 1. A client using tampons who has a fever, is dizzy, and hypotensive might have toxic shock syndrome.

The nurse is caring for four newborns who have recently been admitted to the newborn nursery. Which labor event puts the newborn at risk for an alteration of health? 1. The infant's mother has group B streptococcal (GBS) disease. 2. The infant's mother had an IV of lactated Ringer's solution. 3. The infant's mother had a labor that lasted 12 hours. 4. The infant's mother had a cesarean birth with her last child.

Answer: 1 Explanation: 1. A common cause of neonatal distress is early-onset group B streptococcal (GBS) disease. Infected mothers transmit GBS infection to their infants during labor and birth. All infants of mothers identified as at risk should be assessed and observed for signs and symptoms of sepsis.

The client's Pap smear result is ASC-US. Which statement is the best way for the nurse to explain this ASC-US result? 1. "Abnormal squamous cells of undetermined significance." 2. "Cancer has invaded the upper cervix." 3. "High-grade squamous intraepithelial lesion (HSIL), which includes CIN." 4. "The focus of the Pap smear is the detection of high-risk pregnancy." 5. "The cervical cells are abnormal and the reason why is severe dysplasia and carcinoma in situ."

Answer: 1 Explanation: 1. ASC-US stands for abnormal squamous cells of undetermined significance. Preferred management is HPV testing; if positive, refer for colposcopy; if negative, repeat HPV co-testing in 3 years.

Abdominal hysterectomy is generally recommended for which condition? 1. Severe endometriosis 2. Removal of the ovaries 3. Suspected or confirmed cancer removal 4. Abnormal uterine bleeding

Answer: 1 Explanation: 1. Abdominal hysterectomy is recommended for severe endometriosis.

The nurse is caring for a postpartal client of Hmong descent who immigrated to the United States 5 years ago. The client asks for the regular hospital menu because American food tastes best. The nurse assesses this response to be related to which of the following cultural concepts? 1. Acculturation 2. Ethnocentrism 3. Enculturation 4. Stereotyping

Answer: 1 Explanation: 1. Acculturation (assimilation) is the correct assessment because the client adapted to a new cultural norm in terms of food choices.

The client reports relief from headaches when she rubs the temples on each side of the head. The nurse understands that this is a form of which of the following? 1. Acupressure 2. Acupuncture 3. Reflexology 4. Hydrotherapy

Answer: 1 Explanation: 1. Acupressure uses pressure from the fingers and thumbs to stimulate pressure points to relieve symptoms.

The 12-year-old client reports that menarche occurred 5 months ago. She has had bleeding every day this month, and is very worried. The nurse should explain that the most common cause of this bleeding is which of the following? 1. Dysfunctional uterine bleeding (DUB) 2. Diabetes mellitus (DM) 3. Pregnancy 4. Von Willebrand's disease

Answer: 1 Explanation: 1. Adolescents often experience DUB during the first 2 years following menarche due to hypothalamic immaturity after menarche.

Appropriate nursing interventions for the application of erythromycin ophthalmic ointment (Ilotycin) include which of the following? 1. Massaging eyelids gently following application 2. Irrigating eyes after instillation 3. Using a syringe to apply ointment 4. Instillation is in the upper conjunctival surface of each eye

Answer: 1 Explanation: 1. After administration, the nurse massages the eyelid gently to distribute the ointment.

The nurse is preparing a class on breastfeeding for pregnant women in their first trimester. The women are from a variety of cultural backgrounds, and all speak English well. Which statement should the nurse include in this presentation? 1. "Although some cultures believe colostrum is not good for the baby, it provides protection from infections and helps the digestive system to function." 2. "Some women are uncomfortable with exposing their breasts to nurse their infant, but it really isn't a big deal. You will get used to it." 3. "No religion prescribes a feeding method, so you all can choose whatever method makes the most sense to you." 4. "In most cultures, it is culturally acceptable to speak about intimate matters in front of their families."

Answer: 1 Explanation: 1. Although it is true that some cultures believe colostrum to be unhealthy, colostrum helps to protect the infant from disease and illness.

The nurse is teaching the parents of an infant with an inborn error of metabolism how to care for the infant at home. What information does teaching include? 1. Specially prepared formulas 2. Cataract problems 3. Low glucose concentrations 4. Administration of thyroid medication

Answer: 1 Explanation: 1. An afflicted PKU infant can be treated by a special diet that limits ingestion of phenylalanine. Special formulas low in phenylalanine, such as Lofenalac, Minafen, and Albumaid XP, are available.

The nurse is scheduling an ultrasound for a 51-year-old client with a suspected uterine tumor. The nurse includes in the client's teaching that the most common benign uterine tumor seen in women in their 50s is which of the following? 1. Fibroid tumor 2. Fibroadenoma 3. Fibrocystic tumor 4. Lymphoma

Answer: 1 Explanation: 1. By the age of 50, 70% of Caucasian women and 80% of African American women have fibroids.

During a counseling session on natural family planning techniques, the nurse explains that cervical mucus at the time of ovulation should be of what consistency? 1. Egg white appearance and stretchable 2. Opaque and acidic 3. High in leukocytes 4. Lacking in quantity

Answer: 1 Explanation: 1. Cervical mucus at the time of ovulation has an "egg white" appearance and is known as fertile mucus; it is friendly to sperm because it assists passage through the cervix and uterus up into the fallopian tubes.

A pregnant client at 24 weeks' gestation is diagnosed with bacterial vaginosis. Her doctor orders Flagyl to treat the problem. What would be appropriate education for the nurse to provide? 1. The client must be careful to observe for signs of preterm labor. 2. The client should advise her partner to seek therapy as soon as possible. 3. The main side effect of the medication is a large amount of vaginal discharge. 4. A repeat culture should be taken 2 weeks after completing the therapy.

Answer: 1 Explanation: 1. Clients with bacterial vaginosis are at risk for preterm labor.

The nurse who is taking a sexual history from a client should do which of the following? 1. Ask questions that the client can answer with "yes" or "no." 2. Ask mostly open-ended questions. 3. Have the client fill out a comprehensive questionnaire and review it after the client leaves. 4. Try not to make much direct eye contact.

Answer: 2 Explanation: 2. Open-ended questions are often useful in eliciting information.

The nurse is teaching a prenatal class about feeding methods. A father-to-be asks the nurse which method, breast or formula, leads to the fastest infant growth and weight gain. Which response by the nurse is best? 1. "In the first 3 to 4 months breastfed babies tend to gain weight faster." 2. "In the first 3 to 4 months there is no difference in weight gain." 3. "In the first 3 to 4 months bottle-fed babies grow faster." 4. "In the first 3 to 4 months growth isn't as important as your comfort with the method."

Answer: 1 Explanation: 1. Exclusively breastfed infants have the same or slightly higher weight gain than their formula-fed and combination-fed peers in the first 3 to 4 months.

The nurse obtains a health history from four clients. To which client should she give priority for teaching about cervical cancer prevention? 1. Age 30, treated for PID 2. Age 25, monogamous 3. Age 20, pregnant 4. Age 27, uses a diaphragm

Answer: 1 Explanation: 1. Exposure to sexually transmitted infections increases the risk of abnormal cell changes and cervical cancer.

A 3-month-old baby who was born at 25 weeks has been exposed to prolonged oxygen therapy. Due to oxygen therapy, the nurse explains to the parents, their infant is at a greater risk for which of the following? 1. Visual impairment 2. Hyperthermia 3. Central cyanosis 4. Sensitive gag reflex

Answer: 1 Explanation: 1. Extremely premature newborns are particularly susceptible to injury of the delicate capillaries of the retina causing characteristic retinal changes known as retinopathy of prematurity (ROP). Judicious use of supplemental oxygen therapy in the premature infant has become the norm.

In planning care for a new family immediately after birth, which procedure would the nurse most likely withhold for 1 hour to allow time for the family to bond with the newborn? 1. Eye prophylaxis medication 2. Drying the newborn 3. Vital signs 4. Vitamin K injection

Answer: 1 Explanation: 1. Eye prophylaxis medication instillation may be delayed up to 1 hour after birth to allow eye contact during parent-newborn bonding.

The nurse is teaching a woman about her menstrual cycle. Which is the most important change that happens during the follicular phase of the menstrual cycle? 1. Maturation of the primordial follicle 2. Multiplication of the fimbriae 3. Secretion of human chorionic gonadotropin 4. Growth of the endometrium

Answer: 1 Explanation: 1. Follicle-stimulating hormone is elevated during the follicular phase, and the primordial follicle matures.

The nurse is teaching a class to the community on mind-based therapies. A class participant gives an example of a friend with leukemia who was taught by her complementary therapist to concentrate on making antibodies that will fight and kill the cancer cells in the bloodstream. How would the nurse identify this technique? 1. Guided imagery 2. Qigong 3. Biofeedback 4. Homeopathy

Answer: 1 Explanation: 1. Guided imagery is a state of intense, focused concentration used to create compelling mental images and is useful in imagining a desired effect.

The nurse obtains a health history from four clients. To which client should she give priority for teaching about cervical cancer prevention? 1. Age 37, multiple partners 2. Age 22, abstains from sexual intercourse 3. Age 32, pregnant with twins 4. Age 27, uses female condom

Answer: 1 Explanation: 1. Having multiple partners increases the client's risk of contracting sexually transmitted infections, including possible exposure to human papilloma virus (HPV). Contracting HPV increases risk of abnormal cervical cell changes and cervical cancer.

The nurse is teaching nursing students about the different kind of hepatitis. Which statement is the nurse likely to make? 1. Hepatitis A and B have vaccines to prevent them. 2. Hepatitis A, B, and C have vaccines to prevent them. 3. Hepatitis C, D, and E are all bloodborne. 4. Hepatitis A, C, and E are all fecal-oral contamination.

Answer: 1 Explanation: 1. Hepatitis A and B are the only two types of hepatitis that have vaccines.

The nurse is caring for the newborn of a diabetic mother whose blood glucose level is 39 mg/dL. What should the nurse include in the plan of care for this newborn? 1. Offer early feedings with formula or breast milk. 2. Provide glucose water exclusively. 3. Evaluate blood glucose levels at 12 hours after birth. 4. Assess for hypothermia.

Answer: 1 Explanation: 1. IDMs whose serum glucose falls below 40 mg/dL should have early feedings with formula or breast milk (colostrum).

A nurse is caring for a client admitted preoperatively for a bilateral mastectomy. Which statement indicates that this client is still in the shock phase of adjustment to her diagnosis? 1. "I can't understand why this is happening to me." 2. "I am so happy that my daughter graduates from high school this year." 3. "I know that breast cancer is now part of my life." 4. "I can't believe that I need to lose both of my breasts."

Answer: 1 Explanation: 1. In the shock phase, a client makes statements such as this.

The nurse is performing an assessment on an infant whose mother states that she feeds the infant in a supine position by propping the bottle. Based on this information, what would the nurse include in the assessment? 1. Otoscopic exam of the eardrum 2. Bowel sounds 3. Vital signs 4. Skin assessment

Answer: 1 Explanation: 1. Infants who bottle feed in a supine position have an increased risk of otitis media and dental caries in the older infant.

The nurse is admitting a Mexican woman scheduled for a cholecystectomy. The nurse uses a cultural assessment tool during the admission. Which question would be most important for the nurse to ask? 1. "What other treatments have you used for your abdominal pain?" 2. "In what country were you were born?" 3. "When you talk to family members, how close do you stand?" 4. "How would you describe your role within your family?"

Answer: 1 Explanation: 1. Knowing what other treatments the client has used for pain is most important because some traditional or folk remedies include the use of herbs, which can have medication interactions.

A client comes to the clinic complaining of difficulty urinating, flu-like symptoms, genital tingling, and blister-like vesicles on the upper thigh and vagina. She denies having ever had these symptoms before. The medication the physician is most likely to order would be: 1. Oral acyclovir 2. Ceftriaxone IM 3. Azithromycin p.o. 4. Penicillin G IM

Answer: 1 Explanation: 1. Malaise, dysuria, and tingling or painful vesicles are indicative of a primary herpes simplex outbreak. Acyclovir treats herpes.

The nurse working on a postoperative gynecology unit has to be knowledgeable of the psychological concerns of many of her clients. What will these concerns most often include? 1. Feelings of loss, grief, and anger 2. Feelings of euphoria and happiness 3. Feelings of control and calmness 4. Feelings of cheerfulness and satisfaction

Answer: 1 Explanation: 1. Many gynecological procedures have the potential to involve loss, so grieving, anger, sadness, and loss of control are just a few of the feelings the woman might experience.

The nurse is caring for the newborn of a diabetic mother. Which of the following should be included in the nurse's plan of care for this newborn? 1. Offer early feedings. 2. Administer an intravenous infusion of glucose. 3. Assess for hypercalcemia. 4. Assess for hyperbilirubinemia immediately after birth.

Answer: 1 Explanation: 1. Newborns of diabetic mothers may benefit from early feeding as they are extremely valuable in maintaining normal metabolism and lowering the possibility of such complications as hypoglycemia and hyperbilirubinemia.

The nurse has completed a community education session on growth patterns of infants. Which statement by a participant indicates that additional teaching is needed? 1. "Newborns should regain their birth weight by 1 week of age." 2. "Breastfed and formula-fed babies have different growth rates." 3. "Formula-fed infants regain their birth weight earlier than breastfed infant." 4. "Healthcare providers consider breastfeeding to be the 'gold standard' for neonatal nutrition."

Answer: 1 Explanation: 1. Newborns should gain at least 10 g/kg/day and be back to birth weight no later than day 14 of life.

In planning care for the fetal alcohol syndrome (FAS) newborn, which intervention would the nurse include? 1. Allow extra time with feedings. 2. Assign different personnel to the newborn each day. 3. Place the newborn in a well-lit room. 4. Monitor for hyperthermia.

Answer: 1 Explanation: 1. Newborns with fetal alcohol syndrome have feeding problems. Because of their feeding problems, these infants require extra time and patience during feedings.

The mother of a premature newborn questions why a gavage feeding catheter is placed in the mouth of the newborn and not in the nose. What is the nurse's best response? 1. "Most newborns are nose breathers." 2. "The tube will elicit the sucking reflex." 3. "A smaller catheter is preferred for feedings." 4. "Most newborns are mouth breathers."

Answer: 1 Explanation: 1. Orogastric insertion is preferable to nasogastric because most infants are obligatory nose breathers.

What information should the nurse include when teaching a new mother how to successfully bottle-feed her newborn? 1. Proper dilution of powdered formula is essential to provide adequate nutrition. 2. Keep formula at room temperature for at least 4 hours to warm it, instead of microwaving it. 3. Use enough water to dilute the nutrient and calorie density so the infant will drink more formula. 4. Freeze newly prepared formula for up to 3 months.

Answer: 1 Explanation: 1. Parents should be instructed to follow the directions on the formula can label precisely as written.

A postpartum client calls the nursery to report that her newborn's umbilical cord stump is draining, and has a foul odor. What is the nurse's best response? 1. "Take your newborn to the pediatrician." 2. "Cover the cord stump with gauze." 3. "Apply Betadine around the cord stump." 4. "This is normal during healing."

Answer: 1 Explanation: 1. Parents should check cord each day for any odor, oozing of greenish yellow material, or reddened areas around the cord. They should report to healthcare provider any signs of infection.

The nurse is completing the discharge teaching of a young first-time mother. Which statement by the mother requires immediate intervention? 1. "I will put my baby to bed with his bottle so he doesn't get hungry during the night." 2. "My baby will probably have a bowel movement each breastfeeding, and will wet often." 3. "Nursing every 2 to 3 hours is normal, for a total of 8 to 12 feedings every day." 4. "I will drink fenugreek tea from my grandmother to prevent my milk from coming in."

Answer: 1 Explanation: 1. Putting a baby to bed with a propped bottle is a choking hazard, and should never be done.

When assessing a client asking about birth control, the nurse knows that the client would not be a good candidate for Depo-Provera (DMPA) if which of the following is true? 1. She wishes to get pregnant within 3 months. 2. She is a nursing mother. 3. She has a vaginal prolapse. 4. She weighs 200 pounds.

Answer: 1 Explanation: 1. Return of fertility after use may be delayed for an average of 10 months.

The nurse is interviewing an adolescent client. The client reports a weight loss of 50 pounds over the last 4 months, and reports running at least 5 miles per day. The client asserts that her menarche was 5 years ago. Her menses are usually every 28 days, but her last menstrual period was 4 months ago. The client denies any sexual activity. Which is the best statement for the nurse to make? 1. "Your lack of menses might be related to your rapid weight loss." 2. "It is common and normal for runners to stop having any menses." 3. "Increase your intake of iron-rich foods to reestablish menses." 4. "Adolescents rarely have regular menses, even if they used to be regular."

Answer: 1 Explanation: 1. Secondary amenorrhea can be caused by rapid weight loss, including the development of the eating disorders anorexia and bulimia. Runners with low body fat might have irregular menses, but amenorrhea is not a normal condition.

The nurse seeing a client just diagnosed with Chlamydia trachomatis knows that which client is at greatest risk for the infection? 1. 16-year-old sexually active girl, using no contraceptive 2. 22-year-old mother of two, developed dyspareunia 3. 35-year-old woman on oral contraceptives 4. 48-year-old woman with hot flashes and night sweats

Answer: 1 Explanation: 1. Teens have the highest incidence of sexually transmitted infections, especially chlamydia. A client not using contraceptives is not using condoms, which decrease the risk of contracting a STI.

The parents of a newborn have just been told their infant has tetralogy of Fallot. The parents do not seem to understand the explanation given by the physician. What statement by the nurse is best? 1. "With this defect, not enough of the blood circulates through the lungs, leading to a lack of oxygen in the baby's body." 2. "The baby's aorta has a narrowing in a section near the heart that makes the left side of the heart work harder." 3. "The blood vessels that attach to the ventricles of the heart are positioned on the wrong sides of the heart." 4. "Your baby's heart doesn't circulate blood well because the left ventricle is smaller and thinner than normal."

Answer: 1 Explanation: 1. Tetralogy of Fallot is a cyanotic heart defect that comprises four abnormalities: pulmonary stenosis, ventricular septal defect, overriding aorta, and right ventricle hypertrophy. The severity of symptoms depends on the degree of pulmonary stenosis, the size of the ventricular septal defect, and the degree to which the aorta overrides the septal defect.

A client describes breast swelling and tenderness. What piece of data would be most important for the nurse to gather initially? 1. Timing of the symptoms 2. Birth control method 3. Method of breast self-examination 4. Diet history

Answer: 1 Explanation: 1. The breast undergoes regular cyclical changes in response to hormonal stimulation. The nurse will want to determine when the swelling and tenderness occur within the menstrual cycle.

The nurse provides a couple with education about the consequences of not treating chlamydia, and knows they understand when they make which statement? 1. "She could become pregnant." 2. "She could have severe vaginal itching." 3. "He could get an infection in the tube that carries the urine out." 4. "It could cause us to develop a rash."

Answer: 3 Explanation: 3. Chlamydia is a major cause of nongonococcal urethritis (NGU) in men.

A client in the women's clinic asks the nurse, "How is the cervical mucus method of contraception different from the rhythm method?" The appropriate response by the nurse is that the cervical mucus method is which of the following? 1. More effective for women with irregular cycles 2. Not acceptable to women of many different religions 3. Harder to work with than is the rhythm method 4. Requires an artificial substance or device

Answer: 1 Explanation: 1. The cervical mucus method (Billings Ovulation Method) can be used by women with irregular cycles.

A nurse is providing contraceptive counseling to a perimenopausal client in a monogamous relationship. What comment by the client indicates that further teaching is necessary? 1. "The calendar method is the most reliable method for me to use." 2. "If I use the IUC, I will be at minimal risk for pelvic inflammatory disease." 3. "I should still use birth control, even though I had only three periods last year." 4. "The contraceptive skin patch contains both estrogen and progesterone."

Answer: 1 Explanation: 1. The client who believes the calendar method is the most reliable method requires more teaching. The menstrual cycle of perimenopausal women is irregular, and it is difficult to determine safe and unsafe times.

The nurse is working with a woman who is undergoing chemotherapy for breast cancer. The client states, "First, the cancer seemed unreal. Now I feel like I can cope." What is the nurse's best response? 1. "Women with breast cancer often go through several stages of adjustment." 2. "Women with breast cancer cope better than their partners cope." 3. "Women with breast cancer seek multiple opinions before starting treatment." 4. "Women with breast cancer become angry after treatment begins."

Answer: 1 Explanation: 1. The course of adjustment confronting the woman with cancer has been described in four phases: shock, reaction, recovery, and reorientation. The client's statement indicates shock followed by reaction.

A client using oral contraceptives tells the nurse that her family is complete, and she now desires permanent contraception. Which statement should the nurse include in teaching this client about sterilization options? 1. "Essure becomes effective 3 months after insertion." 2. "Vasectomy is effective immediately after the procedure." 3. "Tubal ligation cannot be performed until the client is age 35." 4. "Oral contraception should be taken until menopause."

Answer: 1 Explanation: 1. The insertion of Essure creates a tissue response that results in tubal occlusion in about 3 months.

The nurse is teaching a group of menopausal women about the signs and symptoms of menopause and how they can get relief. One of the main concerns of the group is vaginal dryness and difficult intercourse. What is the reason the nurse will give for this? 1. The loss of cervical gland function leads to dryness of the mucous membranes of the vagina. 2. The vaginal pH increases, and the number of Doderlein's bacilli decreases. 3. The uterine lining thins and the muscle layer atrophies. 4. The labia shrink and lose their pigmentation.

Answer: 1 Explanation: 1. The loss of cervical gland function leads to dryness of the mucous membranes of the vagina.

The OB-GYN nurse knows that the most common shape for the female pelvis is which of the following? 1. Gynecoid type 2. Android type 3. Anthropoid type 4. Platypelloid type

Answer: 1 Explanation: 1. The most common female pelvis is the gynecoid type. The inlet is rounded with the anteroposterior diameter a little shorter than the transverse diameter.

The nurse in the OB-GYN office is explaining the benefits and risks associated with Essure to a client and her husband. What statement made by the client indicates that additional education is needed? 1. "A stainless steel microinsert is placed into the distal section of one fallopian tube." 2. "The Essure method of permanent sterilization requires no surgical incision." 3. "The steel microinserts create a benign tissue response that occludes the fallopian tubes." 4. "Women allergic to nickel should consult their physician before placement."

Answer: 1 Explanation: 1. The stainless steel microinsert is placed in the proximal section of both fallopian tubes.

The pregnant client at 41 weeks is scheduled for labor induction. She asks the nurse whether induction is really necessary. What response by the nurse is best? 1. "Babies can develop postmaturity syndrome, which increases their chances of having complications after birth." 2. "When infants are born 2 or more weeks after their due date, they have meconium in the amniotic fluid." 3. "Sometimes the placenta ages excessively, and we want to take care of that problem before it happens." 4. "The doctor wants to be proactive in preventing any problems with your baby if he gets any bigger."

Answer: 1 Explanation: 1. The term postmaturity applies to the infant who is born after 42 completed weeks of gestation and demonstrates characteristics of postmaturity syndrome.

The nurse teaching a class on reproductive anatomy knows that no further instruction is needed when a student shows an understanding of the pelvic cavity divisions by making which statement? 1. "The true pelvis is made up of the sacrum, coccyx, and innominate bones." 2. "The false pelvis consists of the inlet, the pelvic cavity, and the outlet." 3. "The true pelvis is the portion above the pelvic brim." 4. "The relationship between the false pelvis and the fetal head is of paramount importance."

Answer: 1 Explanation: 1. The true pelvis is made up of the sacrum, the coccyx, and the two innominate bones.

The nurse is caring for several pregnant clients. Which client should the nurse anticipate is most likely to have a newborn at risk for mortality or morbidity? 1. 37-year-old, with a history of multiple births and preterm deliveries who works in a chemical factory 2. 23-year-old of low socioeconomic status, unmarried 3. 16-year-old who began prenatal care at 30 weeks 4. 28-year-old with a history of gestational diabetes

Answer: 1 Explanation: 1. This client is at greatest risk because she has multiple risk factors: age over 35, high parity, history of preterm birth, and exposure to chemicals that might be toxic.

A woman is asking the nurse about using the calendar method of contraception. She reports that her last six menstrual cycles were 28, 32, 29, 36, 30, and 27 days long, respectively. Based on this information, when should the nurse tell the client to abstain from intercourse? 1. Days 9-25 2. Days 9-15 3. Days 10-21 4. Days 10-16

Answer: 1 Explanation: 1. To calculate the period of abstinence, the nurse must subtract 18 from the shortest cycle length and 11 from the longest cycle length.

The nurse is caring for a 2-hour-old newborn whose mother is diabetic. The nurse assesses that the newborn is experiencing tremors. Which nursing action has the highest priority? 1. Obtain a blood calcium level. 2. Take the newborn's temperature. 3. Obtain a bilirubin level. 4. Place a pulse oximeter on the newborn.

Answer: 1 Explanation: 1. Tremors are a sign of hypocalcemia. Diabetic mothers tend to have decreased serum magnesium levels at term. This could cause secondary hypoparathyroidism in the infant.

In caring for the premature newborn, the nurse must assess hydration status continually. Assessment parameters should include which of the following? Note: Credit will be given only if all correct choices and no incorrect choices are selected. Select all that apply. 1. Volume of urine output 2. Weight 3. Blood pH 4. Head circumference 5. Bowel sounds

Answer: 1, 2 Explanation: 1. In order to assess hydration status, volume of urine output must be evaluated. 2. In order to assess hydration status, the infant's weight must be evaluated.

The nurse is evaluating the outcomes of nursing care for a woman with a urinary tract infection. Which of the following does the nurse include in the evaluation? Note: Credit will be given only if all correct and no incorrect choices are selected. Select all that apply. 1. The client implements self-care measures for prevention. 2. The client completed the prescribed antibiotics. 3. The client knows self-care measures for worsening symptoms. 4. The client states that UTIs are controlled, not cured. 5. The client knows that cranberry juice can help prevent UTIs.

Answer: 1, 2 Explanation: 1. In the evaluation, the nurse should determine whether the client implements self-care measures to help prevent recurrent UTI as part of her personal routine. 2. Evaluation includes whether the client completed her prescribed course of antibiotic therapy.

Why is it important for the nurse to understand the type of family that a client comes from? Note: Credit will be given only if all correct choices and no incorrect choices are selected. Select all that apply. 1. Family structure can influence finances. 2. Some families choose to conceive or adopt without a life partner. 3. The nurse can anticipate which problems a client will experience based on the type of family the client has. 4. Understanding if the client's family is nuclear or blended will help the nurse teach the client the appropriate information. 5. The values of the family will be predictable if the nurse knows what type of family the client is a part of.

Answer: 1, 2 Explanation: 1. Single-parent families often face difficulties because the sole parent may lack social and emotional support, need assistance with childrearing issues, and face financial strain. 2. In the single mother by choice family, the mother is typically older, college-educated, and financially stable and has contemplated pregnancy significantly prior to conceiving.

The nurse teaches a group of young women that self-care measures for dysmenorrhea include which of the following actions? Note: Credit will be given only if all correct and no incorrect choices are selected. Select all that apply. 1. Taking vitamins B and E 2. Decreasing salt intake 3. Using cold packs as needed 4. Using intermittent exercise 5. Taking FSH replacement

Answer: 1, 2 Explanation: 1. Some nutritionists suggest that vitamins B and E help relieve the discomforts associated with menstruation.

The nurse in the emergency department is admitting a 22-year-old woman who complains of watery diarrhea, dizziness, and vomiting. She is in the middle of her period, and has a tampon in place. The nurse suspects toxic shock syndrome, and assesses for the manifestations of which symptom? Note: Credit will be given only if all correct and no incorrect choices are selected. Select all that apply. 1. Fever 2. Rash 3. Desquamation of skin 4. Bloating 5. Urinary frequency and urgency

Answer: 1, 2, 3 Explanation: 1. A fever presents initially. 2. A rash on the trunk presents initially. 3. The fever and rash on the trunk present initially, followed by desquamation of the skin, especially the palms and soles, which usually occurs 1 to 2 weeks after the onset of symptoms.

What are the three functions of the fallopian tubes? Note: Credit will be given only if all correct choices and no incorrect choices are selected. Select all that apply. 1. Provide transport for the ovum from the ovary to the uterus 2. Serve as a warm, moist, nourishing environment for the ovum or zygote 3. Secrete large amounts of estrogens 4. Provide a site for fertilization to occur 5. Support and protect the pelvic contents

Answer: 1, 2, 4 Explanation: 1. The fallopian tubes provide transport for the ovum from the ovary to the uterus. 2. The fallopian tubes serve as a warm, moist, nourishing environment for the ovum or zygote. 4. The fallopian tubes provide a site for fertilization to occur.

In assessing a family, the community nurse uses a family assessment tool, which provides an organized framework to collect data concerning which of the following? Note: Credit will be given only if all correct and no incorrect choices are selected. Select all that apply. 1. Access to laundry and grocery facilities 2. Access to health care 3. Sharing of religious beliefs and values 4. Acculturation to traditional lifestyles 5. Ability to include a new spouse into the family unit

Answer: 1, 2, 3 Explanation: 1. Access to laundry, grocery, and recreational facilities is a means of meeting the physical, emotional, and spiritual needs of members, which is part of the family assessment tool. 2. Access to health care is a means of meeting the physical, emotional, and spiritual needs of members, which is part of the family assessment tool. 3. Sharing of religious beliefs and values is a means of meeting the physical, emotional, and spiritual needs of members, which is part of the family assessment tool.

) In teaching a group of adolescents, the nurse discusses which risk factors for cardiovascular disease (CVD) in women? Note: Credit will be given only if all correct and no incorrect choices are selected. Select all that apply. 1. Being over 55 and postmenopausal 2. Using cigarettes and tobacco 3. Being overweight 4. Having a low cholesterol level 5. Having an active lifestyle

Answer: 1, 2, 3 Explanation: 1. Being over the age of 55 and postmenopausal increases the risk of CVD. 2. The use of cigarettes and tobacco increases the risk of CVD. 3. Being overweight or obese increases the risk of CVD.

During the assessment phase of a family, the community nurse recognizes that culture influences childrearing and childbearing in which of the following ways? Note: Credit will be given only if all correct and no incorrect choices are selected. Select all that apply. 1. Beliefs about the importance of children 2. Beliefs and attitudes about pregnancy 3. Norms regarding infant feeding 4. Acculturation is important in rearing children 5. Time orientation to the future is very important

Answer: 1, 2, 3 Explanation: 1. Culture influences beliefs about the importance of children. 2. Culture influences attitudes about pregnancy and the right vs. the obligation of women to bear children. 3. Culture influences infant feeding norms and practices.

What issues should the nurse consider when counseling a client on contraceptive methods? Note: Credit will be given only if all correct and no incorrect choices are selected. Select all that apply. 1. Cultural perspectives on menstruation and pregnancy 2. Effectiveness of the method 3. Future childbearing plans 4. Whether the client is a vegetarian 5. Age at menarche

Answer: 1, 2, 3 Explanation: 1. Decisions about contraception should be made voluntarily with full knowledge of advantages, disadvantages, effectiveness, side effects, contraindications, and long-term effects. Many outside factors influence this choice, including cultural practices, religious beliefs, personality, cost, effectiveness, availability, misinformation, practicality of method, and self-esteem. 2. Decisions about contraception should be made voluntarily with full knowledge of advantages, disadvantages, effectiveness, side effects, contraindications, and long-term effects. Many outside factors influence this choice, including cultural practices, religious beliefs, personality, cost, effectiveness, availability, misinformation, practicality of method, and self-esteem. 3. Decisions about contraception should be made voluntarily with full knowledge of advantages, disadvantages, effectiveness, side effects, contraindications, and long-term effects. Many outside factors influence this choice, including cultural practices, religious beliefs, personality, cost, effectiveness, availability, misinformation, practicality of method, and self-esteem.

The nurse is providing health education to a group of young people. When teaching about the prevention of sexually transmitted infections (STIs), the nurse will teach which concepts? Note: Credit will be given only if all correct and no incorrect choices are selected. Select all that apply. 1. Decision-making skills in refusing intercourse 2. How to reduce high-risk behaviors 3. That Pap smears might be needed more often 4. The safety of oral sex 5. Use of petroleum-based lubricants with condoms

Answer: 1, 2, 3 Explanation: 1. Effective prevention and control of STIs is based on planning ahead, review of decision-making skills, and developing strategies to refuse sex. 2. Effective prevention and control of STIs is based on the reduction of high-risk behaviors. Use of recreational drugs and alcohol can increase sexual risk taking. 3. The presence of a genital infection can lead to an abnormal Pap smear. Women with certain infections should have more frequent Pap tests.

The nurse is educating a group of women about the manifestations of benign breast conditions. What information topics will the nurse include? Note: Credit will be given only if all correct and no incorrect choices are selected. Select all that apply. 1. Masses rapid in appearance or disappearance 2. Galactorrhea is common 3. Galactorrhea is seen with some psychiatric medications 4. Ulcerations or skin erosions are possible 5. An increased breast size with dimpling

Answer: 1, 2, 3 Explanation: 1. Fluctuations in size and rapid appearance or disappearance of breast masses are common in fibrocystic breasts. 2. Galactorrhea is a benign change often associated with hormone therapy. 3. Galactorrhea is often seen in women who have nursed, who have fibrocystic changes in the breast, or are on some types of psychiatric medication.

In assessing a new family coming to the clinic, the nurse determines they are an extended kin family because the family exhibits what as characteristics of an extended kin network family? Note: Credit will be given only if all correct and no incorrect choices are selected. Select all that apply. 1. A sharing of a social support network 2. Each family establishes their own sources of goods and services 3. Elderly parents share housing 4. Children are members of two nuclear families 5. A sharing of goods and services

Answer: 1, 5 Explanation: 1. Extended kin family networks share a social support network. 5. Extended kin family networks share goods and services.

What are some of the advantages and disadvantages of formula-feeding that a nurse should discuss with new parents? Note: Credit will be given only if all correct choices and no incorrect choices are selected. Select all that apply. 1. The nutritional value of formula depends on the proper preparation/dilution. 2. There is a potential for bacterial contamination during preparation and storage. 3. Both parents can participate in positive parent-infant interaction during feeding. 4. Refrigeration is not necessary if preparing more than one bottle at a time. 5. Formula has higher levels of essential fatty acids, lactose, cystine, and cholesterol than does breast milk.

Answer: 1, 2, 3 Explanation: 1. Formula composition does not vary unless the instructions for preparation and dilution are not followed. 2. Bacteria can be introduced if bottles and nipples are not properly cleaned. 3. If a mother has chosen not to breastfeed, then the father can be involved with bottle feedings from the start.

In working with immigrants in an inner-city setting, the nurse recognizes that acculturation of immigrants often brings with it which of the following benefits? Note: Credit will be given only if all correct and no incorrect choices are selected. Select all that apply. 1. Improved socioeconomic status 2. Use of preventive care services 3. Better nutrition 4. Increase in substance abuse over time 5. More physician visits due to language barriers

Answer: 1, 2, 3 Explanation: 1. Improvement of socioeconomic status is a benefit of acculturation in the United States. 2. Acculturation of immigrants increases the likelihood that the family members will use preventive health services. 3. Improved socioeconomic status leads to better nutrition and access to health care.

The client in the first trimester of pregnancy tells the nurse she regularly sees a massage therapist to help with pain in her shoulders and neck. The nurse tells her that massage has which of the following added advantages? Note: Credit will be given only if all correct and no incorrect choices are selected. Select all that apply. 1. Increasing circulation 2. Reducing anxiety 3. Promoting a sense of well-being 4. Eliminating energy blockages 5. Allowing coordination of breathing and moving

Answer: 1, 2, 3 Explanation: 1. Massage therapy has been shown to increase circulation. 2. Massage therapy has been shown to reduce anxiety. 3. Massage therapy has been shown to promote a sense of well-being.

The nurse is counseling a pregnant woman who intends to see a naturopathic physician. The nurse tells the woman that she can expect education on which of the following? Note: Credit will be given only if all correct and no incorrect choices are selected. Select all that apply. 1. Clinical nutrition 2. Botanical medicine 3. Lifestyle modification 4. Use of "like to cure like" 5. The role of yin and yang

Answer: 1, 2, 3 Explanation: 1. Naturopathic physicians teach their clients about clinical nutrition. 2. The use of botanical medicine is a part of naturopathy. 3. Lifestyle modification is a part of naturopathy.

The nurse is educating a client on intrauterine contraception (IUC). Which advantages of IUC will the nurse include? Note: Credit will be given only if all correct and no incorrect choices are selected. Select all that apply. 1. It is effective, continuous contraception. 2. It is relatively inexpensive over time. 3. It does not need to be inserted immediately prior to coitus. 4. Small amounts of estrogen are released daily. 5. It slightly increases the risk of ectopic pregnancy.

Answer: 1, 2, 3 Explanation: 1. One advantage of IUC is a high rate of effectiveness. 2. One advantage of IUC is that it is inexpensive over time. 3. It is non-coitus-related contraception.

The nurse is writing a care plan for a client with abnormal uterine bleeding (AUB). Which goals does the nurse include? Note: Credit will be given only if all correct and no incorrect choices are selected. Select all that apply. 1. Preventing or treating anemia 2. Preventing endometrial hyperplasia 3. Restoring quality of life 4. Conducting pelvic exams monthly 5. Analyzing liver and kidney function

Answer: 1, 2, 3 Explanation: 1. One goal for the client with AUB is to prevent or treat anemia. 2. One goal for the client with AUB is to prevent endometrial hyperplasia. 3. One goal for a client with AUB is to restore quality of life.

The nurse has assessed a client who is to undergo gynecological surgery. Upon what will the nurse base the nursing diagnosis? Note: Credit will be given only if all correct and no incorrect choices are selected. Select all that apply. 1. The client's attitudes toward motherhood 2. The client's body image 3. The client's attitudes about sexuality 4. The client's long-term risk reduction 5. The client's use of acupuncture

Answer: 1, 2, 3 Explanation: 1. Reproductive surgery may also be seen as a threat to femininity in any social or cultural group that emphasizes childbearing and motherhood. 2. Surgery to alter or remove reproductive organs may be perceived as a threat to self-concept. Body image is affected whenever a body part is lost. 3. Many women fear postoperative changes such as masculinization, weight gain, loss of sexuality, and permanent loss of the ability to have a child.

The nurse at a women's clinic is planning a class on premenstrual dysphoric disorder (PMDD). The nurse includes in the education information about what medications that are shown to be effective for PMDD? Note: Credit will be given only if all correct and no incorrect choices are selected. Select all that apply. 1. Fluoxetine hydrochloride (Prozac) 2. Sertraline hydrochloride (Zoloft) 3. Paroxeline CR (Paxil CR) 4. Hyoscyamine (Anaspaz) 5. Promethazine (Phenergan)

Answer: 1, 2, 3 Explanation: 1. Selective serotonin inhibitors such as fluoxetine hydrocholoride (Prozac) have been found to be effective in controlling PMDD. 2. Setraline hydrochloride (Zoloft) is a selective serotonin inhibitor and has been found to be effective in controlling PMDD. 3. Paroxeline CR (Paxil CR) is a selective serotonin inhibitor and has been found to be effective in controlling PMDD.

The nurse is discharging a client after hospitalization for pelvic inflammatory disease (PID). Which statements indicate that teaching was effective? Note: Credit will be given only if all correct and no incorrect choices are selected. Select all that apply. 1. "I might have infertility because of this infection." 2. "It is important for me to finish my antibiotics." 3. "Tubal pregnancy could occur after PID." 4. "My PID was caused by a yeast infection." 5. "I am going to have an IUD placed for contraception."

Answer: 1, 2, 3 Explanation: 1. Women sometimes become infertile because of scarring in the fallopian tubes as a result of the inflammation of PID. 2. Antibiotic therapy should always be completed when a client is diagnosed with any infection. 3. The tubal scarring that occurs from tubal inflammation during PID can prevent a fertilized ovum from passing through the tube into the uterus, causing an ectopic or tubal pregnancy.

When teaching the new mother about the composition of breast milk, the nurse explains that the fat content can range from 30 to 50 grams/liter. Which factors affect the fat content of breast milk? Note: Credit will be given only if all correct choices and no incorrect choices are selected. Select all that apply. 1. Maternal parity 2. Duration of pregnancy 3. Stage of lactation 4. Time of day 5. Vitamin C intake

Answer: 1, 2, 3, 4 Explanation: 1. Multiparous mothers produce milk with a lower content of fatty acids than primigravidas. 2. The milk of a mother who delivers a preterm infant has a greater concentration of DHA and ARA than does the milk of a mother who gives birth to a full-term infant. 3. Phospholipids and cholesterol levels are higher in colostrum than in mature milk, although overall fat content is higher in mature breast milk than in colostrum. 4. Fat content is generally higher in the evening and lower in the early morning.

A client is diagnosed with a cystocele. The nurse actively listens as the client talks about which treatment choice to make. The treatment options available to the client are which of the following? Note: Credit will be given only if all correct and no incorrect choices are selected. Select all that apply. 1. Kegel exercises 2. Vaginal pessaries 3. Surgical intervention 4. Estrogen 5. Fine-needle aspiration

Answer: 1, 2, 3, 4 Explanation: 1. One treatment option is to use Kegel exercises. 2. A treatment choice the client has is to have a pessary or ring inserted. 3. Surgical intervention is one approach to helping the client with a cystocele. 4. Estrogen may improve the condition of vaginal mucous membranes, especially in menopausal women.

What are the nurse's responsibilities when teaching the new mother about infant feeding? Note: Credit will be given only if all correct choices and no incorrect choices are selected. Select all that apply. 1. The nurse should be well informed about infant nutrition and feeding methods. 2. The nurse should provide accurate and consistent information. 3. The nurse should use each interaction to support the parents and promote the family's sense of confidence. 4. The nurse should familiarize the mother with information about community resources that might be helpful after discharge. 5. The nurse should aggressively promote breastfeeding, even if the parents have decided to bottle-feed their infant.

Answer: 1, 2, 3, 4 Explanation: 1. The nurse should provide information on maternal and infant nutrition, fluid requirements, and feeding methods. 2. The nurse should provide information that is accurate and consistent. Nurses have a responsibility to educate parents. 3. Every interaction is a chance to educate and support the new parents in learning how to care for the newborn. Nurses have a responsibility to educate parents. 4. It is important that parents receive verbal and written instructions and community resource information to which they can later refer.

The nurse teaching a group of new mothers and explains that barrier methods of contraception are appropriate if which of the following conditions exist? Note: Credit will be given only if all correct and no incorrect choices are selected. Select all that apply. 1. Just started an oral contraceptive 2. Is opposed to systemic medications 3. Is lactating 4. Is unconcerned about using contraception 5. Does not smoke and is perimenopausal

Answer: 1, 2, 3, 5 Explanation: 1. A backup method of contraception is needed for a period of time when beginning oral contraceptives. 2. Opposition to taking systemic medications is a reason to use barriers for contraception. 3. Using a barrier method of contraception is appropriate in the early postpartum period or when lactating. 5. Being perimenopausal and smoking make barrier contraception an appropriate choice.

The nurse is evaluating the outcomes of nursing care given a female client with a sexually transmitted infection (STI). Which of the following will the nurse verify about the client? Note: Credit will be given only if all correct and no incorrect choices are selected. Select all that apply. 1. Had an identified infection 2. Could identify the mode of transmission 3. Would never cope with the infection 4. Could recognize the symptoms of an STI 5. Would not disclose the STI to her partner

Answer: 1, 2, 4 Explanation: 1. The infection must be identified and cured, if possible. If not, supportive therapy is provided. 2. The client and her partner should be able to describe the infection, its method of transmission, its implications, and the therapy. 4. The client must be able to recognize the symptoms of an STI.

When planning care for a client who has undergone an episiotomy, it would be important for the nurse to include a goal that addresses the need for pain relief of which part of the body? 1. Mons pubis 2. Perineal body 3. Labia minora 4. Hymen

Answer: 2 Explanation: 2. During the last part of labor, the perineal body thins out until it is just a few centimeters thick. This tissue is often the site of an episiotomy or lacerations during childbirth.

The public health nurse is working with a student nurse. The student nurse asks which of the six groups of people they have seen today are considered to be families. How should the nurse respond? Note: Credit will be given only if all correct choices and no incorrect choices are selected. Select all that apply. 1. "The married heterosexual couple without children" 2. "The gay couple with two adopted children" 3. "The unmarried heterosexual couple with two biological children" 4. "The lesbian couple not living together that have no children" 5. "The married heterosexual couple with three children, living with grandparents"

Answer: 1, 2, 3, 5 Explanation: 1. Families take many forms in today's society. The basis for people to be considered a family is a commitment to one another and the sharing of responsibilities, chores, and expenses. A couple without children is still a family. 2. Families take many forms in today's society. The basis for people to be considered a family is a commitment to one another and the sharing of responsibilities, chores, and expenses. Gay and lesbian families are those in which two or more people who share a same-sex orientation live together, or in which a gay or lesbian single parent rears a child. 3. Families take many forms in today's society. The basis for people to be considered a family is a commitment to one another and the sharing of responsibilities, chores, and expenses. A family may be formed without a legal marriage. 5. Families take many forms in today's society. The basis for people to be considered a family is a commitment to one another and the sharing of responsibilities, chores, and expenses. Extended family members, including parents or grandparents, will often live with their adult children or grandchildren, creating intergenerational families.

The culturally sensitive nurse will assess cultural attitudes and beliefs about contraception, which can include which of the following? Note: Credit will be given only if all correct and no incorrect choices are selected. Select all that apply. 1. Gender inequities 2. Religion 3. Deference to authority figures 4. Devaluation of large families 5. Attitudes about bleeding

Answer: 1, 2, 3, 5 Explanation: 1. Gender inequities may prohibit some Arab, Latina, and Eastern Indian women from seeking out or using a contraceptive method unless their husbands do not object. 2. The Roman Catholic Church considers all artificial methods of contraception unacceptable. 3. Deference to authority figures is not uncommon in traditional Chinese, Arab, Latina, and East Indian women, especially if the nurse is male. 5. Attitudes toward bleeding affect a woman's duties to her family and partner. Vaginal bleeding may be seen as "unclean" by Muslim and Orthodox Jewish women. Any contraceptive method that involves irregular bleeding might not be acceptable. Among women who feel a monthly period is necessary, any method that ultimately causes amenorrhea would not be acceptable.

A nurse is teaching contraception to a group of college students during a health class. What reason will the nurse include as to why couples might choose to use contraception? Note: Credit will be given only if all correct and no incorrect choices are selected. Select all that apply. 1. Couples use contraception to avoid pregnancy. 2. Couples use contraception to space future pregnancies. 3. Couples use contraception to gain control over the number of children conceived. 4. Couples use contraception to avoid decisions about the number of children to have. 5. Couples use contraception to avoid contracting sexually transmitted diseases.

Answer: 1, 2, 3, 5 Explanation: 1. This is the main reason for the use of contraception. 2. Contraception is used to space children. 3. Contraception is used to control the number of children that a couple desires. 5. Some types of birth control will avoid sexually transmitted diseases—for example, some of the barrier methods.

The nurse in the OB-GYN clinic has been seeing a client through her menopausal stage of life. The nurse assesses psychological concerns if the client makes which statement? Note: Credit will be given only if all correct and no incorrect choices are selected. Select all that apply. 1. "I feel so lost with all of the kids away from home." 2. "I wish I were younger, and could have a baby." 3. "Although my parents are aging, they travel a lot." 4. "I don't think I am a good wife anymore." 5. "I really enjoy being able to go out when I want."

Answer: 1, 2, 4 Explanation: 1. Adjustment to an "empty nest" is a psychological concern during menopause. 2. Some women express disappointment in approaching this time of their lives, whereas others might see it as a positive transition that offers freedom from menses or concern about contraception. 4. Numerous personal factors influence a woman's ability to transition and cope with these changes, such as self-concept.

The OB-GYN nurse knows that the proliferative phase of the menstrual cycle includes which changes? Note: Credit will be given only if all correct and no incorrect choices are selected. Select all that apply. 1. Thin, clear cervical mucus 2. Estrogen peaks just before ovulation 3. No ferning pattern of cervical mucus 4. A pH change to 7.5 5. Epithelium warps into folds

Answer: 1, 2, 4 Explanation: 1. Cervical mucus at ovulation becomes thin, clear, watery, and more alkaline, making it more favorable to spermatozoa. 2. Estrogen peaks just before ovulation in the proliferative phase of the menstrual cycle. 4. The cervical mucosa pH increases from below 7 to 7.5 at the time of ovulation.

The client with polycystic ovarian syndrome (PCOS) has been prescribed metformin (Glucophage). The nurse tells the client that the medication will do which of the following? Note: Credit will be given only if all correct choices and no incorrect choices are selected. Select all that apply. 1. "Decrease your excessive hair growth." 2. "Make it easier to lose weight." 3. "Increase your acne." 4. "Improve your chances of pregnancy." 5. "Make your menstrual periods irregular."

Answer: 1, 2, 4 Explanation: 1. Polycystic ovarian syndrome (PCOS) treatment with metformin decreases hirsutism. 2. Polycystic ovarian syndrome (PCOS) treatment with metformin improves weight loss success. 4. Polycystic ovarian syndrome (PCOS) treatment with metformin increases ovulation and therefore menstrual regularity and fertility.

The nurse tells a mother that the doctor is preparing to circumcise her newborn. The mother expresses concern that the infant will be uncomfortable during the procedure. The nurse explains that the physician will numb the area before the procedure. Additional methods of comfort often used during the procedure include which of the following? Note: Credit will be given only if all correct choices and no incorrect choices are selected. Select all that apply. 1. Providing a pacifier 2. Stroking the head 3. Restraining both arms and legs 4. Talking to the infant 5. Giving the infant a sedative before the procedure

Answer: 1, 2, 4 Explanation: 1. Providing a pacifier is an accepted method of soothing during the circumcision. 2. Stroking the head is an accepted method of soothing during the circumcision. 4. Talking to the infant is an accepted method of soothing during the circumcision.

The nurse is teaching a group of perimenopausal women about treatment choices for their symptoms. The nurse will include which information? Note: Credit will be given only if all correct and no incorrect choices are selected. Select all that apply. 1. Importance of continuing contraception 2. Benefits of menstrual regulation with hormones 3. Increase in vaginal dryness with hormones 4. Decrease in acne and hirsutism 5. Increase in endometrial cancer risk

Answer: 1, 2, 4 Explanation: 1. Thirty-eight percent of pregnancies for women ages 40 and older are unplanned; some form of contraception is needed. 2. Regulation of menses with effective contraception is a benefit to the perimenopausal woman. 4. Women on hormones might experience less acne and hirsutism.

The nurse assessing a 47-year-old client who is perimenopausal includes which important topics? Note: credit will be given only if all correct and no incorrect choices are selected. Select all that apply. 1. Vasomotor symptoms 2. A decrease in vaginal lubrication 3. Pregnancy not being an option 4. Mood changes that occur 5. An increase in the libido

Answer: 1, 2, 4 Explanation: 1. Women need to know that vasomotor symptoms occur. It might be important to investigate other possible causes of the vasomotor symptoms. 2. Women need to know that a decrease in vaginal lubrication occurs, and that water-soluble lubricants should be used. 4. Mood changes and irritability occur, and the woman might experience more forgetfulness.

The nurse is discussing the use of contraception with a client who has just become sexually active. What factors should the nurse include when educating the client on contraceptive methods? Note: Credit will be given only if all correct and no incorrect choices are selected. Select all that apply. 1. Contraindications in the client's health history 2. Religious or moral beliefs 3. Partner's belief in the effectiveness of the choice 4. Personal preferences to use method 5. Future childbearing plans

Answer: 1, 2, 4, 5 Explanation: 1. Decisions about contraception should take into consideration any contraindications the client might have. 2. Religious or moral beliefs often impact which choices are acceptable. 4. Personal preferences need to be considered when deciding on a contraceptive method. 5. Plans for future children should be considered before determining whether sterilization should be performed.

The nurse is caring for a newborn with full fontanelles and "setting sun" eyes. Which nursing interventions should be included in the care plan? Note: Credit will be given only if all correct choices and no incorrect choices are selected. Select all that apply. 1. Measure head circumference daily. 2. Assess for bulging fontanelles. 3. Avoid position changes. 4. Watch for signs of infection. 5. Use a gel pillow under the head.

Answer: 1, 2, 4, 5 Explanation: 1. The infant has congenital hydrocephalus. The nurse should measure and plot occipital-frontal baseline measurements, then measure head circumference once a day. 2. The infant has congenital hydrocephalus. Fontanelles should be checked for bulging and sutures for widening. 4. Infants with hydrocephalus are prone to infection. 5. The infant has congenital hydrocephalus. The enlarged head should be supported with a gel pillow.

The nurse is teaching a group of clients about risk factors for osteoporosis. The nurse will include which of the following risk factors in the teaching? Note: Credit will be given only if all correct and no incorrect choices are selected. Select all that apply. 1. Menopause at an early age increases bone loss 2. A family history of osteoporosis 3. A lifetime of high calcium intake 4. Having an active lifestyle 5. A vitamin D deficiency

Answer: 1, 2, 5 Explanation: 1. Abnormal absence of menses and early onset of menopause increase the risk of osteoporosis. 2. Family history of osteoporosis, especially a maternal hip fracture, increases the risk of osteoporosis. 5. Vitamin D deficiency increases the risk of osteoporosis.

In learning about Duvall's life-cycle stages ascribed to traditional families, the nursing student recognizes that developmental tasks of each stage include which of the following? Note: Credit will be given only if all correct and no incorrect choices are selected. Select all that apply. 1. Adjusting to new roles as mother and father 2. Working out authority and socialization roles with the school 3. Becoming a single parent with custodial responsibilities 4. Becoming a couple and dating 5. Adjusting to the loss of a spouse

Answer: 1, 2, 5 Explanation: 1. Adjusting to new roles as mother and father occurs in Stage II, which describes childbearing families with infants. 2. Working out authority and socialization roles with schools occurs in Stage IV, which describes families with school-age children. 5. Stage VIII includes adjusting to the loss of a spouse.

The 22-year-old client is scheduled for her first gynecologic examination. What can the nurse do to make the client more comfortable during this exam? Note: Credit will be given only if all correct choices and no incorrect choices are selected. Select all that apply. 1. Create a trusting atmosphere. 2. Show the client what the speculum looks like. 3. Avoid telling the client what the exam involves. 4. Ask the client why she has delayed her first Pap test this long. 5. Provide a mirror for the client.

Answer: 1, 2, 5 Explanation: 1. It is important to create a trusting atmosphere and incorporate practices that help the client maintain a sense of control. 2. Show the client all of the equipment to be used. 5. Provide a mirror to increase learning about anatomy and to create a trusting atmosphere.

Which of the following are potential disadvantages to breastfeeding? Note: Credit will be given only if all correct choices and no incorrect choices are selected. Select all that apply. 1. Pain with breastfeeding 2. Leaking milk 3. Equal feeding responsibilities with fathers 4. Vaginal wetness 5. Embarrassment

Answer: 1, 2, 5 Explanation: 1. Nipple tenderness is the most common source of discomfort and is usually related to improper positioning and/or not obtaining a proper attachment of the infant on the breast. Pain can also be related to engorgement or infection. 2. Some women will leak milk when their breasts are full and it is nearly time to breastfeed again or whenever they experience let-down. Mothers should be given reassurance that this problem diminishes over time. 5. Some mothers feel uncomfortable about breastfeeding because they are modest or may feel embarrassed because our society views breasts as sexual objects and/or an unfriendly social environment makes it difficult to breastfeed in public. This is not an easy issue to overcome.

Ovarian hormones include which of the following? Note: Credit will be given only if all correct and no incorrect choices are selected. Select all that apply. 1. Estrogens 2. Progesterone 3. Parathyroid hormone 4. Luteinizing hormone 5. Testosterone

Answer: 1, 2, 5 Explanation: 1. Ovarian hormones include the estrogens, progesterone, and testosterone. 2. Ovarian hormones include the estrogens, progesterone, and testosterone. 5. Ovarian hormones include the estrogens, progesterone, and testosterone.

A new mother is questioning the nurse about the advantages of breastfeeding her newborn. Which information should the nurse include in the teaching session? Note: Credit will be given only if all correct choices and no incorrect choices are selected. Select all that apply. 1. Breast milk has immunological advantages, including varying degrees of protection from bacterial and viral infections. 2. Breastfeeding has been shown to increase maternal-infant attachment. 3. Breastfeeding can be initially supplemented with bottle feedings so that the father does not feel left out of the infant's care. 4. Breastfeeding often causes nipple tenderness, and may be discouraged until healing occurs. 5. Breastfeeding provides a psychologic advantage to the mother, who derives satisfaction knowing that she is providing her infant with the optimal nutritional start in life.

Answer: 1, 2, 5 Explanation: 1. Secretory IgA, an immunoglobulin present in colostrum and mature breast milk, has antiviral, antibacterial, and antigenic-inhibiting properties. 2. Early breastfeeding can enhance maternal-infant bonding. 5. The mother's sense of accomplishment in being able to satisfy her baby's needs for nourishment and comfort can be a tremendous source of personal satisfaction.

Which statements by a breastfeeding class participant indicate that teaching by the nurse was effective? Note: Credit will be given only if all correct choices and no incorrect choices are selected. Select all that apply. 1. "Breastfed infants get more skin-to-skin contact and sleep better." 2. "Breastfeeding raises the level of a hormone that makes me feel good." 3. "Breastfeeding is complex and difficult, and I probably won't succeed." 4. "Breastfeeding is worthwhile, even if it costs more overall." 5. "Breastfed infants have fewer digestive and respiratory illnesses."

Answer: 1, 2, 5 Explanation: 1. Skin-to-skin contact after birth helps the baby maintain his or her body temperature, helps with self-regulation, increases maternal oxytocin levels, helps the mother to notice subtle feeding cues, and promotes bonding. 2. Hormones of lactation promote maternal feelings and sense of well-being. 5. This is a true statement. The immunologic advantages of human milk include varying degrees of protection from respiratory tract and gastrointestinal tract infections.

The nurse is administering erythromycin (Ilotycin) ointment to a newborn. What factors are associated with administration of this medication? Note: Credit will be given only if all correct choices and no incorrect choices are selected. Select all that apply. 1. The medication should be instilled in the lower conjunctival sac of each eye. 2. The eyelids should be massaged gently to distribute the ointment. 3. The medication must be given immediately after delivery. 4. The medication does not cause any discomfort to the infant. 5. The medication can interfere with the baby's ability to focus.

Answer: 1, 2, 5 Explanation: 1. Successful eye prophylaxis requires that the medication be instilled in the lower conjunctival sac of each eye. 2. After administration, the nurse massages the eyelid gently to distribute the ointment. 5. Eye prophylaxis medication can cause chemical conjunctivitis, which gives the newborn some discomfort and can interfere with the baby's ability to focus on the parents' faces.

Which of the following activities allows the nurse to provide individualized parent teaching on the maternal-infant unit? Note: Credit will be given only if all correct choices and no incorrect choices are selected. Select all that apply. 1. Teach by example and role modeling when caring for the newborn in the client's room. 2. Teach at every opportunity, even during the night shift, if the occasion arises. 3. Teach using newborn care videos and group classes. 4. Teach using the 24-hour educational television channels in the client's room. 5. Teach using one-to-one instruction while in the client's room.

Answer: 1, 2, 5 Explanation: 1. The nurse can be an excellent role model for families. Teaching by example is a very effective way to teach infant care. 2. One-to-one teaching while the nurse is in the client's room is shown to be the most effective educational model. 5. One-to-one teaching while the nurse is in the client's room is the most effective educational model.

To maintain a healthy temperature in the newborn, which of the following actions should be taken? Note: Credit will be given only if all correct choices and no incorrect choices are selected. Select all that apply. 1. Keep the newborn's clothing and bedding dry. 2. Reduce the newborn's exposure to drafts. 3. Do not use the radiant warmer during procedures. 4. Do not wrap the newborn. 5. Encourage the mother to snuggle with the newborn under blankets.

Answer: 1, 2, 5 Explanation: 1. To maintain a healthy temperature in the newborn, keep the newborn's clothing and bedding dry. 2. To maintain a healthy temperature in the newborn, reduce the newborn's exposure to drafts. 5. To maintain a healthy temperature in the newborn, encourage the mother to snuggle with the newborn under blankets.

The nurse working in a women's clinic is training a recent graduate of nursing school who has been hired. The experienced nurse explains that nurses caring for women of all ages must be which of the following? Note: Credit will be given only if all correct and no incorrect choices are selected. Select all that apply. 1. Aware of their own feelings 2. Judgmental when discussing sexuality 3. Aware of personal values and attitudes 4. Minimally knowledgeable about reproduction 5. Willing to discuss sexuality only one-on-one

Answer: 1, 3 Explanation: 1. Nurses must be aware of their own feelings. 3. Nurses must develop an awareness of their own values and attitudes about sexuality so that they can be more sensitive and objective when they encounter the values and beliefs of others.

The nurse assessing a 50-year-old female client at an orthopedic center asks about the use of complementary and alternative therapies. Which of the following are among those women often try during menopause? Note: Credit will be given only if all correct and no incorrect choices are selected. Select all that apply. 1. Vitamin and mineral supplements 2. Increasing caffeine intake 3. Soy and red clover 4. Selective estrogen receptor modulators 5. Salmon calcitonin

Answer: 1, 3 Explanation: 1. Vitamin and mineral supplements, especially calcium and vitamins D, E, and B complex, are used to control the symptoms of menopause. 3. Soy and red clover have shown to be effective for some women in controlling their menopausal symptoms.

The nurse encourages a new mother to feed the newborn as soon as the newborn shows interest. The nurse bases this recommendation on which benefits of early feedings? Note: Credit will be given only if all correct choices and no incorrect choices are selected. Select all that apply. 1. Early feedings stimulate peristalsis. 2. Colostrum is thinner than mature milk. 3. Early feedings enhance maternal-infant bonding. 4. Early feedings promote the passage of meconium. 5. Colostrum contains a high number of calories.

Answer: 1, 3, 4 Explanation: 1. Early breastfeeding stimulates the newborn's peristalsis. 3. Early breastfeeding enhances maternal-infant bonding. 4. Early breastfeeding promotes the passage of meconium.

The nurse initiates newborn admission procedures and evaluates the newborn's need to remain under observation by assessing which of the following? Note: Credit will be given only if all correct choices and no incorrect choices are selected. Select all that apply. 1. Respiratory rate 2. Skin texture 3. Airway clearance 4. Ability to feed 5. Head weight

Answer: 1, 3, 4 Explanation: 1. The nurse initiates newborn admission procedures and evaluates the newborn's need to remain under observation by assessing vital signs (body temperature, heart rate, respiratory rate). 3. The nurse initiates newborn admission procedures and evaluates the newborn's need to remain under observation by assessing airway clearance. 4. The nurse initiates newborn admission procedures and evaluates the newborn's need to remain under observation by assessing ability to feed.

Many newborns exposed to HIV/AIDS show signs and symptoms of disease within days of birth that include which of the following? Note: Credit will be given only if all correct choices and no incorrect choices are selected. Select all that apply. 1. Swollen glands 2. Hard stools 3. Smaller than average spleen and liver 4. Rhinorrhea 5. Interstitial pneumonia

Answer: 1, 4, 5 Explanation: 1. Signs that may be seen in the early infancy period include swollen glands. 4. Signs that may be seen in the early infancy period include rhinorrhea. 5. Signs that may be seen in the early infancy period include interstitial pneumonia.

The nurse is teaching the parents of a newborn who has been exposed to HIV how to care for the newborn at home. Which instructions should the nurse emphasize? Note: Credit will be given only if all correct choices and no incorrect choices are selected. Select all that apply. 1. Use proper hand-washing technique. 2. Provide three feedings per day. 3. Place soiled diapers in a sealed plastic bag. 4. Cleanse the diaper changing area with a 1:10 bleach solution after each diaper change. 5. Take the temperature rectally.

Answer: 1, 3, 4 Explanation: 1. The nurse should instruct the parents on proper hand-washing technique. 3. The nurse should instruct parents to that soiled diapers are to be placed in plastic bags, sealed, and disposed of daily. 4. The nurse should instruct parents that the diaper-changing areas should be cleaned with a 1:10 dilution of household bleach after each diaper change.

A nurse is instructing nursing students about the procedure for vitamin K administration. What information should be included? Note: Credit will be given only if all correct choices and no incorrect choices are selected. Select all that apply. 1. Gently massage the site after injection. 2. Use a 22-gauge, 1-inch needle. 3. Inject in the vastus lateralis muscle. 4. Cleanse the site with alcohol prior to injection. 5. Inject at a 45-degree angle.

Answer: 1, 3, 4 Explanation: 1. The nurse would remove the needle and massage the site with an alcohol swab. 3. Vitamin K is given intramuscularly in the vastus lateralis muscle. 4. Before injecting, the nurse must clean the newborn's skin site for the injection thoroughly with a small alcohol swab.

The nurse is preparing to give an injection of vitamin K to a newborn. Which considerations would be appropriate? Note: Credit will be given only if all correct choices and no incorrect choices are selected. Select all that apply. 1. Administer a dose of 0.5 to 1 mg within 1 hour of birth. 2. Administer the injection subcutaneously. 3. Use a 25-gauge, 5/8-inch needle for the injection. 4. Protect the medication bottle from light. 5. Give vitamin K prior to a circumcision procedure.

Answer: 1, 3, 4, 5 Explanation: 1. 0.5 to 1 mg is the correct dosage for vitamin K. 3. 25-gauge, 5/8-inch needle is the right size needle to use. 4. Vitamin K must be kept away from light. 5. A prophylactic injection of vitamin K1 is given to prevent hemorrhage, which can occur because of low prothrombin levels in the first few days of life.

Which assessment findings would lead the nurse to suspect that a newborn might have a congenital heart defect? Note: Credit will be given only if all correct choices and no incorrect choices are selected. Select all that apply. 1. Cyanosis 2. Heart murmur 3. Bradycardia 4. Low urinary outputs 5. Tachypnea

Answer: 1, 3, 4, 5 Explanation: 1. Central cyanosis is defined as a visible, blue discoloration of the skin caused by decreased oxygen saturation levels and is a common manifestation of a cardiac defect. 3. The signs of congestive heart failure include tachycardia, not bradycardia. 4. The signs of congestive heart failure include low urinary output. 5. The signs of congestive heart failure include tachypnea.

) The nurse is caring for a prenatal client. Reviewing the client's pregnancy history, the nurse identifies risk factors for an at-risk newborn, including which of the following? Note: Credit will be given only if all correct choices and no incorrect choices are selected. Select all that apply. 1. The mother's low socioeconomic status 2. Maternal age of 26 3. Mother's exposure to toxic chemicals 4. More than three previous deliveries 5. Maternal hypertension

Answer: 1, 3, 4, 5 Explanation: 1. Low socioeconomic status is associated with at-risk newborns. 3. Exposure to environmental dangers, such as toxic chemicals is associated with at-risk newborns. 4. Maternal factors such as multiparity are associated with at-risk newborns. 5. Preexisting maternal conditions, such as heart disease, diabetes, hypertension, hyperthyroidism, and renal disease are associated with at-risk newborns.

The nurse explains the functions of the male reproductive organs to a client. Which correct functions will the nurse include? Note: Credit will be given only if all correct and no incorrect choices are selected. Select all that apply. 1. Ejaculatory ducts move semen and seminal fluid. 2. The adrenal gland is the major source of testosterone. 3. The vas deferens ends before reaching the prostate gland. 4. Sertoli's cells nourish spermatozoa. 5. The testes house seminiferous tubules.

Answer: 1, 5 Explanation: 1. Ejaculatory ducts provide a passageway for semen and seminal fluid into the urethra. 5. The testes house seminiferous tubules and the gonads.

The nurse is providing discharge teaching to the parents of a newborn. The nurse should instruct the parents to notify the healthcare provider in case of which of the following? Note: Credit will be given only if all correct choices and no incorrect choices are selected. Select all that apply. 1. More than one episode of forceful vomiting. 2. More than 6 to 10 wet diapers per day. 3. A bluish discoloration of the skin with or without a feeding. 4. Refusal of two feedings in a row. 5. Development of eye drainage.

Answer: 1, 3, 4, 5 Explanation: 1. More than one episode of forceful vomiting or frequent vomiting over a 6-hour period should be reported to the healthcare provider. 3. Cyanosis (bluish discoloration of skin) with or without a feeding is a cause for concern, and should be reported to the healthcare provider immediately. 4. Refusal of two feedings in a row should be reported to the healthcare provider. 5. The infant should not have eye drainage after discharge and this condition should be reported to the healthcare provider.

The nurse is caring for an infant of a diabetic mother. Which potential complications would the nurse consider in planning care for this newborn? Note: Credit will be given if all correct choices and no incorrect choices are selected. Select all that apply. 1. Tremors 2. Hyperglycemia 3. Hyperbilirubinemia 4. Respiratory distress syndrome 5. Birth trauma

Answer: 1, 3, 4, 5 Explanation: 1. Tremors are a clinical sign of hypocalcemia. 3. Hyperbilirubinemia is caused by slightly decreased extracellular fluid volume, which increases the hematocrit level. 4. Respiratory distress syndrome (RDS) is a complication that occurs more frequently in newborns of diabetic mothers whose diabetes is not well controlled. 5. Because most IDMs are macrosomic, trauma may occur during labor and vaginal birth resulting in shoulder dystocia, brachial plexus injuries, subdural hemorrhage, cephalohematoma, and asphyxia.

The nurse working in a women's clinic recognizes that which clients are most at risk for developing vulvovaginal candidiasis? Note: Credit will be given only if all correct and no incorrect choices are selected. Select all that apply. 1. Clients using antibiotics 2. Immunocompetent clients 3. Pregnant clients 4. Multiparous clients 5. Diabetic clients

Answer: 1, 3, 5 Explanation: 1. A predisposing factor to yeast infections includes the use of antibiotics. 3. Predisposing factors to yeast infections include pregnancy. 5. Predisposing factors to yeast infections include glycosuria and diabetes mellitus.

A nurse is teaching a group of college students at a Catholic university about natural family planning (NFP). Teaching was successful if the students say natural family planning is which of the following? Note: Credit will be given only if all correct and no incorrect choices are selected. Select all that apply. 1. "It is free, safe, and acceptable according to our religion." 2. "It includes breastfeeding for 1 year." 3. "It is useful in helping us plan pregnancies." 4. "It allows us to safely have intercourse during our fertile days." 5. "It does not involve the use of artificial substances or devices."

Answer: 1, 3, 5 Explanation: 1. NFP is free, safe, and acceptable to many whose religious beliefs prohibit other methods. 3. NFP is useful in helping couples to plan a pregnancy. 5. NFP involves no artificial substances or devices.

The nurse is caring for the newborn of a drug-addicted mother. Which assessment findings would be typical for this newborn? Note: Credit will be given only if all correct choices and no incorrect choices are selected. Select all that apply. 1. Hyperirritability 2. Decreased muscle tone 3. Exaggerated reflexes 4. Low pitched cry 5. Transient tachypnea

Answer: 1, 3, 5 Explanation: 1. Newborns born to drug-addicted mothers exhibit hyperirritability. 3. Newborns born to drug-addicted mothers exhibit exaggerated reflexes. 5. Newborns born to drug-addicted mothers exhibit transient tachypnea.

The nurse recognizes the importance of the interaction between the nervous and endocrine systems in the female reproductive cycle. The interaction involves which of the following? Note: Credit will be given only if all correct and no incorrect choices are selected. Select all that apply. 1. Hypothalamus 2. Adrenal cortex 3. Ovaries 4. Thyroid 5. Anterior pituitary

Answer: 1, 3, 5 Explanation: 1. The female reproductive cycle is controlled by complex interactions between the nervous and endocrine systems and their target tissues. These interactions involve the hypothalamus. 3. The female reproductive cycle is controlled by complex interactions between the nervous and endocrine systems and their target tissues. These interactions involve the ovaries. 5. The female reproductive cycle is controlled by complex interactions between the nervous and endocrine systems and their target tissues. These interactions involve the anterior pituitary.

A client who was raped is in the emergency department. She asks the nurse what her alternatives are for emergency contraception. The nurse tells her she can receive which of the following treatments? Note: Credit will be given only if all correct and no incorrect choices are selected. Select all that apply. 1. Take levonorgestrel and ethinyl estradiol. 2. Have Essure implanted. 3. Have a laparoscopic tubal ligation. 4. Take levonorgestrel alone, but in a high dose. 5. Take high doses of male hormones.

Answer: 1, 4 Explanation: 1. Preparations for EC include high doses of levonorgestrel and ethinyl estradiol (Yuzpe regimen). 4. Preparations for EC include high doses of levonorgestrel (Plan B).

A mother and her newborn are being discharged 2 days after delivery. The general discharge instructions provided by the nurse include which of the following? Note: Credit will be given only if all correct choices and no incorrect choices are selected. Select all that apply. 1. Always place the infant in a supine position in the crib. 2. Support the infant's head when carrying for the first week or two. 3. Do not allow the baby to fall asleep in someone's arms. 4. Cover the cord stump with a bandage. 5. Use a bulb syringe to suction mucus from the infant's nostrils as necessary

Answer: 1, 5 Explanation: 1. The newborn should be placed on his or her back (supine) for sleeping. 5. During the first few days of life, the newborn has increased mucus, and gentle suctioning with a bulb syringe may be indicated.

A nurse is performing an assessment on a family with a father and mother who both work. What type of family does she record this family as being? 1. A traditional nuclear family 2. A dual-career/dual-earner family 3. An extended family 4. An extended kin family

Answer: 2 Explanation: 2. A dual-career/dual-earner family is characterized by both parents working, by choice or necessity.

A menopausal woman tells her nurse that she experiences discomfort from vaginal dryness during sexual intercourse, and asks, "What should I use as a lubricant?" The nurse should recommend which of the following? 1. Petroleum jelly 2. A water-soluble lubricant 3. Body cream or body lotion 4. Less-frequent intercourse

Answer: 2 Explanation: 2. A water-soluble jelly should be used.

The nurse is caring for a client in the clinic whom she suspects has vaginosis. Which test best determines whether this sexually active woman has the disorder? 1. The observation of mycelia upon direct microscopy in a 10% potassium hydroxide preparation 2. The addition of a 10% potassium hydroxide solution to the vaginal secretions 3. A vaginal pH of less than 4.5 4. A Gram stain positive for the fungus

Answer: 2 Explanation: 2. Adding a 10% potassium hydroxide solution to the vaginal secretion of a client with bacterial vaginosis, called a "whiff" test, produces a fishy smell.

A 7-year-old client tells the nurse that "Grandpa, Mommy, Daddy, and my brother live at my house." The nurse identifies this as what type of family? 1. Binuclear 2. Extended 3. Gay or lesbian 4. Traditional

Answer: 2 Explanation: 2. An extended family consists of a couple who share the house with their parents, siblings, or other relatives.

A new mother who is breastfeeding tells the nurse that her infant is spitting up frequently, has very loose stools and copious gas, and feeds for only short periods of time. The nurse suspects a feeding intolerance and, after questioning the mother about her diet, suggests that she do which of the following? 1. Stop breastfeeding and switch to formula. 2. Eliminate dairy products from her diet. 3. Supplement breastfeeding with a soy-based formula. 4. Offer the baby water between feedings.

Answer: 2 Explanation: 2. Breastfeeding babies may not be allergic to the mother's milk but rather to the cow's milk protein (an antigen) in the mother's milk. By eliminating the culprit (e.g., the bovine protein) from the mother's diet and therefore from the breast milk, the mother can continue to breastfeed, providing optimal nutrition and immune factors to her infant.

Placing the baby at mother's breast facilitates early latch and promotes successful breastfeeding. When should breastfeeding be initiated? 1. 6 to 12 hours after birth 2. Within 1 hour of birth 3. 24 hours after birth 4. 48 hours after birth

Answer: 2 Explanation: 2. Breastfeeding should be initiated within the first hour of life unless medically contraindicated.

The nurse is teaching a community education class on complementary and alternative therapies. To assess learning, the nurse asks, "In traditional Chinese medicine, what is the invisible flow of energy in the body that maintains health and ensures physiologic functioning?" Which answer indicates that teaching was successful? 1. Meridians 2. Chi 3. Yin 4. Yang

Answer: 2 Explanation: 2. Chi is the energy that flows through the body along meridians, or pathways, to maintain health.

The nurse is answering the perimenopausal client's questions about hormone replacement therapy. Which client statement indicates a need for further teaching? 1. "Estrogen therapy will decrease my chances of developing osteoporosis." 2. "If I am taking estrogen therapy, I will not have to worry about my cholesterol being checked." 3. "Osteoporosis is a decrease in bone strength due to bone density and quality." 4. "Bone mass tends to decrease after menopause."

Answer: 2 Explanation: 2. Cholesterol levels should be checked regularly even when normal, so this statement indicates the need for more teaching.

The nurse is assisting a mother to bottle-feed her newborn, who has been crying. The nurse suggests that prior to feeding, the mother should do which of the following? 1. Offer a pacifier 2. Burp the newborn 3. Unwrap the newborn 4. Stroke the newborn's spine and feet

Answer: 2 Explanation: 2. Crying results in increased ingestion of air even before the infant has started feeding. Infants who are very hungry also gulp more air. For these situations, instruct the parents to burp their infant frequently.

The nurse is preparing an education session for women on the prevention of urinary tract infections (UTIs). Which statement should be included? 1. Lower urinary tract infections rarely occur in women. 2. The most common causative organism of cystitis is E. coli. 3. Wiping from back to front after a BM will help prevent a UTI. 4. Back pain often develops with a lower urinary tract infection.

Answer: 2 Explanation: 2. E. coli is present in 75% to 90% of women with UTIs.

A client is concerned about her risk for breast cancer. Following the initial history, the nurse identifies which of the following as a high risk factor for breast cancer? 1. History of late menarche and early menopause 2. Sister who has had breast cancer 3. Mother with fibrocystic breast disease 4. Multiparity

Answer: 2 Explanation: 2. Family history of first-degree relative (mother, sister, or daughter) with breast cancer increases the risk of breast cancer with the number of first-degree relatives with breast cancer.

A mother who is HIV-positive has given birth to a term female. What plan of care is most appropriate for this infant? 1. Test with a HIV serologic test at 8 months. 2. Begin prophylactic AZT (Zidovudine) administration. 3. Provide 4 to 5 large feedings throughout the day. 4. Encourage the mother to breastfeed the child.

Answer: 2 Explanation: 2. For infants, AZT is started prophylactically 2 mg/kg/dose PO every 6 hours beginning as soon after birth as possible and continuing for 6 weeks.

The nurse is admitting a client who is 12 weeks pregnant and an IV drug user. She has had a number of sexual partners, complains of malaise, and has yellow in the eyes, nausea, and vomiting. Having obtained this history, the nurse suspects that the client has which condition? 1. Hepatitis E 2. Hepatitis C 3. Gonorrhea 4. Hepatitis A

Answer: 2 Explanation: 2. Hepatitis C is bloodborne, and found in drug users and those who have multiple sexual partners.

) A client who is in perimenopause is having a number of severe symptoms. The nurse assesses this client and knows the doctor will likely prescribe what to assist in relieving the distress? 1. Calcium and vitamin D 2. A form of hormonal contraception 3. Prescriptive pain medication 4. Antibiotics

Answer: 2 Explanation: 2. Hormonal contraception is the correct answer, as pregnancy can still be a concern, plus the estrogen will relieve other symptoms, such as hot flashes and vaginal dryness.

A nurse teaches newly pregnant clients that if an ovum is fertilized and implants in the endometrium, the hormone the fertilized egg begins to secrete is which of the following? 1. Estrogen 2. Human chorionic gonadotropin (hCG) 3. Progesterone 4. Luteinizing hormone

Answer: 2 Explanation: 2. If the ovum is fertilized and implants in the endometrium, the fertilized egg begins to secrete human chorionic gonadotropin (hCG), which is needed to maintain the corpus luteum.

Which of the following best describes a nuclear family? 1. A family is composed of an unmarried woman who chooses to conceive or adopt without a life partner. 2. Children live in a household with both biologic parents and no other relatives or persons. 3. A couple shares household and childrearing responsibilities with parents, siblings, or other relatives. 4. The head of the household is widowed, divorced, abandoned, or separated or most often the mother remains unmarried.

Answer: 2 Explanation: 2. In the nuclear family, children live in a household with both biologic parents and no other relatives or persons.

The nurse is caring for a newborn who was recently circumcised. Which nursing intervention is appropriate following the procedure? 1. Keep the infant NPO for 4 hours following the procedure. 2. Observe for urine output. 3. Wrap dry gauze tightly around the penis. 4. Clean with cool water with each diaper change.

Answer: 2 Explanation: 2. It is important to observe for the first voiding after a circumcision to evaluate for urinary obstruction related to penile injury and/or edema.

The nurse is providing follow-up education to a client just diagnosed with vaginal herpes. What statement by the client verifies correct knowledge about vaginal herpes? 1. "I should douche daily to prevent infection." 2. "I could have another breakout during my period." 3. "I am more likely to develop cancer of the cervix." 4. "I should use sodium bicarbonate on the lesions to relieve discomfort."

Answer: 2 Explanation: 2. Menstruation seems to trigger recurrences of herpes.

A client who is currently 12 weeks pregnant is seeking information on pregnancy termination. The nurse is teaching the client about the risks involved if she were to take the medication mifepristone. Which adverse signs would the client need to know and report immediately to a physician? 1. Fatigue, lack of appetite, and inability to concentrate 2. Nausea, diarrhea, vomiting, and stomach pain 3. Increase in heart rate, inability to sit still, and lack of appetite 4. Headache, fatigue, and hunger

Answer: 2 Explanation: 2. Nausea, diarrhea, vomiting, and stomach pain are the symptoms that must be reported immediately to the physician.

A nurse is working in a clinic where clients from several cultures are seen. As a first step toward the goal of personal cultural competence, the nurse will do which of the following? 1. Enhance cultural skills. 2. Gain cultural awareness. 3. Seek cultural encounters. 4. Acquire cultural knowledge.

Answer: 2 Explanation: 2. One begins to gain cultural competence by gaining cultural awareness or by gaining an effective and cognitive self-awareness of personal worldview biases, beliefs, etc.

The nurse is preparing an educational session on phenylketonuria for a family whose neonate has been diagnosed with the condition. Which statement by a parent indicates that teaching was effective? 1. "This condition occurs more frequently among Japanese people." 2. "We must be very careful to avoid most proteins to prevent brain damage." 3. "Carbohydrates can cause our baby to develop cataracts and liver damage." 4. "Our baby's thyroid gland isn't functioning properly."

Answer: 2 Explanation: 2. PKU is the inability to metabolize phenylalanine, an amino acid found in most dietary protein sources. Excessive accumulation of phenylalanine and its abnormal metabolites in the brain tissue leads to progressive, irreversible intellectual disability.

To promote infant security in the hospital, the nurse instructs the parents of a newborn to do which of the following? 1. Keep the baby in the room at all times. 2. Check the identification of all personnel who transport the newborn. 3. Place a "No Visitors" sign on the door. 4. Keep the baby in the nursery at all times.

Answer: 2 Explanation: 2. Parent should be instructed to allow only people with proper birthing unit identification to remove the baby from the room. If parents do not know the staff person, they should call the nurse for assistance.

The nurse's response to a client with a history of pelvic inflammatory disease who is trying to get pregnant is based on the knowledge that which condition can contribute to an infertility problem? 1. Hepatitis 2. Postinfection tubal damage 3. Pelvic abscess 4. Tubal infection

Answer: 2 Explanation: 2. Postinfection tubal damage is the most likely cause, as the infection causes tubal damage, which can lead to infertility.

The community nurse is working with poor women who are formula-feeding their infants. Which statement indicates that the nurse's education session was effective? 1. "I should use only soy-based formula for the first year." 2. "I follow the instructions for mixing the powdered formula exactly." 3. "It is okay to add more water to the formula to make it last longer." 4. "The mixed formula can be left on the counter for a day."

Answer: 2 Explanation: 2. Powdered formula is the least expensive type of formula. Parents will need to be briefed on safety precautions during formula preparation and they should be instructed to follow the directions on the formula package label precisely as written.

A woman has been unable to complete a full-term pregnancy because the fertilized ovum failed to implant in the uterus. This is most likely due to a lack of which hormone? 1. Estrogen 2. Progesterone 3. FSH 4. LH

Answer: 2 Explanation: 2. Progesterone is often called the hormone of pregnancy because it inhibits uterine contractions and relaxes smooth muscle to cause vasodilation, allowing pregnancy to be maintained.

The nurse is working with a client whose religious beliefs differ from those of the general population. What is the best nursing intervention to use to meet the specific spiritual needs of this family? 1. Ask how important the client's religious and spiritual beliefs are when making decisions about health care. 2. Show respect while allowing time and privacy for religious rituals. 3. Ask for the client's opinion on what caused the illness. 4. Identify healthcare practices forbidden by religious or spiritual beliefs.

Answer: 2 Explanation: 2. Providing spiritually sensitive care involves determining the current spiritual and religious beliefs and practices that will affect the mother and baby, accommodating these practices where possible, and examining one's own spiritual or religious beliefs to be more aware and able to provide nonjudgmental care.

The nurse is teaching a new mother how to encourage a sleepy baby to breastfeed. Which of the following instructions would not be included in that teaching? 1. Providing skin-to-skin contact 2. Swaddling the newborn in a blanket 3. Unwrapping the newborn 4. Allowing the newborn to feel and smell the mother's breast

Answer: 2 Explanation: 2. Remove the baby's blanket and clothing so that the infant is wearing only a diaper and T-shirt. Babies feed better when they are not bundled, and they can achieve better attachment without the bulk of extra clothing and blankets. Swaddling the newborn has the opposite effect.

The client demonstrates that the nurse's teaching regarding ways to prevent a recurrence of her urinary tract infection was effective when she makes which statement? 1. "I should wipe from back to front after urination." 2. "I should urinate when I feel the urge." 3. "I should try to restrict my intake of fruits." 4. "I should use a diaphragm."

Answer: 2 Explanation: 2. Retention overdistends the bladder, and can lead to infection.

) A client comes to the clinic complaining of severe menstrual cramps. She has never been pregnant, has been diagnosed with ovarian cysts, and has had an intrauterine device (IUD) for 2 years. What is the most likely cause for the client's complaint? 1. Primary dysmenorrhea 2. Secondary dysmenorrhea 3. Menorrhagia 4. Hypermenorrhea

Answer: 2 Explanation: 2. Secondary dysmenorrhea is associated with pathology of the reproductive tract, and usually appears after menstruation has been established. Conditions that most frequently cause secondary dysmenorrhea include ovarian cysts and the presence of an intrauterine device.

The nurse is working with a new mother who delivered yesterday. The mother has chosen to breastfeed her infant. Which demonstration of skill is the best indicator that the client understands breastfeeding? 1. She puts the infant to breast when he is asleep to help wake him up. 2. She takes off her gown to achieve skin-to-skin contact. 3. She leans toward the infant so that he turns his head to access the nipple. 4. The infant is crying when he is brought to the breast.

Answer: 2 Explanation: 2. Skin-to-skin contact after birth helps the baby maintain his or her body temperature, helps with self-regulation, increases maternal oxytocin levels, helps the mother to notice subtle feeding cues, and promotes bonding.

The nurse educator is planning a class about contraception, and includes information about the effects of various contraceptive methods on sexually transmitted diseases. Which statement will the nurse include concerning spermicides? 1. "Spermicides are effective against gonorrhea and chlamydia, but not against HIV." 2. "Spermicides are not effective against sexually transmitted diseases, and can increase a woman's susceptibility to HIV." 3. "Spermicides are effective against HIV, but are not effective against the other sexually transmitted diseases." 4. "Whether or not spermicides are effective against sexually transmitted diseases depends on where the woman is in her cycle."

Answer: 2 Explanation: 2. Spermicides are not effective against any sexually transmitted disease. Spermicides have a negative effect on the integrity of vaginal cells, making them more susceptible.

After a sex education class at a high school, the nurse overhears a student discussing safe sex practices. Which statement indicates that teaching was successful? 1. "I don't have to worry about getting infected if I have oral sex." 2. "Teen women are the group at highest risk for sexually transmitted infections." 3. "The best thing to do if I have sex a lot is to use spermicide each and every time." 4. "Boys get the HIV virus more easily than girls do."

Answer: 2 Explanation: 2. Teens with multiple sex partners are more susceptible to sexually transmitted diseases.

Which of the following functions primarily to provide low-income women and children who are at risk for medical or nutritional problems with nutritious foods to supplement their diets, nutrition education and counseling, and screening and referrals to other health, welfare, and social programs? 1. ABM 2. WIC 3. ILCA 4. LLLI

Answer: 2 Explanation: 2. The Supplemental Nutrition Program for Women, Infants, and Children (WIC) functions primarily to provide low-income women and children who are at risk for medical or nutritional problems with nutritious foods to supplement their diets, nutrition education and counseling, and screening and referrals to other health, welfare, and social programs.

The nurse receives a phone call from a 25-year-old woman experiencing breast tenderness in the week prior to her menses, with palpable breast nodularity, without nipple discharge. What is the best response by the nurse? 1. "Please make an appointment at the breast cancer center as soon as possible." 2. "How much salty food do you regularly consume?" 3. "As long as you don't have nipple discharge, it isn't a serious condition." 4. "Eliminate caffeine and chocolate from your diet."

Answer: 2 Explanation: 2. The client is describing fibrocystic breast changes. A salt restriction with a mild diuretic taken the week before menstrual bleeding often improves the condition.

The nurse is seeing clients in the women's clinic. Which client should be treated with ceftriaxone IM and doxycycline orally? 1. A pregnant client with gonorrhea and a yeast infection 2. A nonpregnant client with gonorrhea and chlamydia 3. A pregnant client with syphilis 4. A nonpregnant client with chlamydia and trichomoniasis

Answer: 2 Explanation: 2. The combined treatment of ceftriaxone IM and doxycycline orally provides dual treatment for gonorrhea and chlamydia, which frequently occur together.

The external and internal female reproductive organs develop and mature in response to what hormones? 1. Adrenocorticotropic hormones (ACTH) 2. Estrogen and progesterone 3. Steroid hormones 4. Luteinizing hormones (LH)

Answer: 2 Explanation: 2. The external and internal female reproductive organs develop and mature in response to estrogen and progesterone.

The female and male reproductive organs are homologous, which means what? 1. They are believed to cause vasoconstriction and muscular contraction 2. They are fundamentally similar in function and structure 3. They are rich in sebaceous glands 4. They are target organs for estrogenic hormones

Answer: 2 Explanation: 2. The female and male reproductive organs are homologous; that is, they are fundamentally similar in function and structure.

A postpartum mother is concerned that her newborn has not had a stool since birth. The newborn is 18 hours old. What is the nurse's best response? 1. "I will call your pediatrician immediately." 2. "Passage of the first stool within 48 hours is normal." 3. "Your newborn might not have a stool until the third day." 4. "Your newborn must be dehydrated."

Answer: 2 Explanation: 2. The first voiding should occur within 24 hours and first passage of stool within 48 hours.

) The nurse has presented a community education class on recommended health screenings for women. Which statement about the Pap smear by a class member indicates that additional teaching is necessary? 1. "It is recommended for women 21 years of age and older." 2. "It diagnoses cervical cancer." 3. "Intercourse at a young age is a risk factor for an abnormal Pap smear." 4. "Detects abnormal cells."

Answer: 2 Explanation: 2. The focus of the Pap smear is the detection of high-grade cervical disease. It does not diagnose cervical cancer.

A nurse is evaluating the diet plan of a breastfeeding mother, and determines that her intake of fruits and vegetables is inadequate. The nurse explains that the nutritional composition of the mother's breast milk can be adversely affected by this nutritional inadequacy. Which strategy should the nurse recommend to the mother? 1. Stop breastfeeding 2. Provide newborn supplements to the newborn 3. Offer whole milk 4. Supplement with skim milk

Answer: 2 Explanation: 2. The mother may continue to breastfeed, but the caregiver may choose to prescribe additional vitamins for the newborn. Vitamins in human milk are influenced by the mother's vitamin intake, general nutritional status, and genetic differences.

The nurse assesses a sleeping 1-hour-old, 39-weeks'-gestation newborn. The assessment data that would be of greatest concern would be which of the following? 1. Temperature 97.9°F 2. Respirations 68 breaths/minute 3. Vital signs stable for only 2 hours 4. Heart rate 156 beats/min

Answer: 2 Explanation: 2. The normal respiratory rate is 30-60 breaths/min; 68 breaths/min could represent a less-than-ideal transition.

The nurse has just assisted the father in bathing the newborn 2 hours after birth. The nurse explains that the newborn must remain in the radiant warmer. This is based on which assessment data? 1. Heart rate 120 2. Temperature 96.8°F 3. Respiratory rate 50 4. Temperature 99.6°F

Answer: 2 Explanation: 2. The nurse rechecks the temperature after the bath and, if it is stable, dresses the newborn in a shirt, diaper, and cap; wraps the baby; and places the baby in an open crib at room temperature. If the baby's axillary temperature is below 36.5°C (97.7°F), the nurse returns the baby to the radiant warmer. The rewarming process should be gradual to prevent the possibility of hyperthermia.

An HIV-positive mother delivered 2 days ago. The infant will be placed in foster care. The nurse is planning discharge teaching for the foster parents on how to care for the newborn at home. Which instructions should the nurse include? 1. Do not add food supplements to the baby's diet. 2. Place soiled diapers in a sealed plastic bag. 3. Wash soiled linens in cool water with bleach. 4. Shield the baby's eyes from bright lights.

Answer: 2 Explanation: 2. The nurse should instruct the parents about proper hand-washing techniques, about proper disposal of soiled diapers, and to wear gloves when diapering.

The nurse teaching a high school class explains that during the menstrual cycle, the endometrial glands begin to enlarge under the influence of estrogen and cervical mucosal changes occur; the changes peak at ovulation. In which phase of the menstrual cycle does this occur? 1. Menstrual 2. Proliferative 3. Secretory 4. Ischemic

Answer: 2 Explanation: 2. The proliferative phase begins when the endometrial glands begin to enlarge under the influence of estrogen and cervical mucosal changes occur; the changes peak at ovulation.

A client is using the contraceptive sponge as a birth control method. What action is important in order to maintain the sponge's effectiveness? 1. Insert the sponge at least 1 hour before intercourse. 2. Thoroughly moisten the sponge with water before inserting. 3. Spermicidal jelly must be inserted at the same time the sponge is inserted. 4. A new sponge must be inserted every time a couple has intercourse.

Answer: 2 Explanation: 2. The sponge is moistened thoroughly with water before use to activate the spermicide.

The nurse is teaching a class on menstruation to young girls. What information would be important for 10-12-year-olds to know? 1. The age they will start having their periods 2. Variations in the age menstruation begins, length of the cycle, and duration of the menses 3. The number of days they will be ill when they have their menses 4. The number of days they will not be able to take part in physical education at school during their menses

Answer: 2 Explanation: 2. These are the issues young girls like to know, as they always question whether they are different from their peers.

The nurse is about to tell a client that her Pap smear result was abnormal. Which statement should the nurse include? 1. "The Pap smear is used to diagnose cervical cancer." 2. "A loop electrosurgical excision procedure (LEEP) is needed." 3. "Colposcopy to further examine your cervix is the next step." 4. "Your cervix needs to be treated with cryotherapy."

Answer: 3 Explanation: 3. Colposcopy is an examination of the cervix through a magnifying device.

The charge nurse is giving an in-service to the orthopedic and gynecology nurses concerning the relationship between estrogen and osteoporosis. Which is a statement the nurse would make concerning this relationship? 1. "Males have a higher incidence of osteoporosis than do females." 2. "Women who experience menopause at a younger age and have less bone mass could have more bone loss." 3. "Estrogen levels affect only bone mass, but they have nothing to with bone strength and bone density." 4. "Osteoporosis puts the client at a decreased

Answer: 2 Explanation: 2. This is a true statement, as women who experience menopause at a younger age and have less bone mass lose the benefit of estrogen for more years.

The nurse is interviewing a 16-year-old client who has been using deodorant tampons during her menses. She comes into the gynecology office complaining of a rash and open sores on her labia and tenderness in the vagina. After obtaining her history, what will the nurse determine is the most likely cause of this client's problem? 1. She had forceful intercourse, which caused the trauma. 2. She is reacting to the deodorant in the tampon. 3. She might be allergic to the underwear she is wearing. 4. She is having a normal reaction to her menses.

Answer: 2 Explanation: 2. This is the correct answer, as women often will react to the deodorant used on pads and tampons.

At birth, an infant weighed 8 pounds 4 ounces. Three days later, the newborn is being discharged. The parents note that the baby now weighs 7 pounds 15 ounces. The nurse explains that the change in the newborn's weight is which of the following? 1. Excessive 2. Within normal limits 3. Less than expected 4. Unusual

Answer: 2 Explanation: 2. This newborn's weight loss is within normal limits. A weight loss of up to 10% for term newborns is considered within normal limits during the first week of life.

The clinic nurse is returning phone calls. Which call should the nurse return first? 1. The call from a 22-year-old reporting that she has menstrual cramps and vomiting every month 2. The call from a 17-year-old asking whether there is a problem with using one tampon for a whole day 3. The call from a 46-year-old mother of a teen wondering if her daughter should be on birth control 4. The call from a 34-year-old requesting information on douching after intercourse

Answer: 2 Explanation: 2. Using a single tampon for an entire day can lead to toxic shock syndrome, a potentially life-threatening condition. This client needs education on the danger of using one tampon longer than 3-6 hours.

The nurse manager in a hospital with a large immigrant population is planning an in-service. Aware of how ethnocentrism affects nursing care, the nurse manager asks, "The belief that one's own values and beliefs are the only or the best values has which of the following results?" 1. It implies newcomers to the United States should adopt the norms and values of the country. 2. It can create barriers to communication through misunderstanding. 3. It leads to an expectation that all clients will exhibit pain the same way. 4. It improves the quality of care provided to culturally diverse client bases.

Answer: 2 Explanation: 2. When the nurse assumes that a client has the same values and beliefs as the nurse, misunderstanding will frequently occur, which in turn can negatively impact nurse-client communication. Ethnocentrism is the conviction that the values and beliefs of one's own cultural group are the best or only acceptable ones.

The nurse teaches a client that luteinizing hormone (LH) is important in the ovarian cycle for which purposes? Note: Credit will be given only if all correct and no incorrect choices are selected. Select all that apply. 1. Proliferation of the endometrial mucosa 2. Ovulation 3. Corpus luteum development 4. Maturation of the ovarian follicle 5. Cyclic changes that allow pregnancy not to occur

Answer: 2, 3 Explanation: 2. During the follicular phase, the primordial follicle matures under the influence of FSH and LH until ovulation occurs. 3. The corpus luteum develops under the influence of LH during the luteal phase.

A client comes to the clinic complaining of a thick, white, tenacious discharge and vulvular soreness. Which medication treatment will the nurse teach the client about? Note: Credit will be given only if all correct and no incorrect choices are selected. Select all that apply. 1. Silver sulfadiazine 2. Metronidazole 3. Clindamycin cream 4. Ceftriaxone sodium 5. Doxycycline

Answer: 2, 3 Explanation: 2. Metronidazole is the preferred treatment for vulvovaginal candidiasis. 3. Although less effective than metronidazole, clindamycin cream can be used to treat vulvovaginal candidiasis.

Which instructions should the nurse include when teaching parents of a newborn about caring for the umbilical cord? Note: Credit will be given only if all correct choices and no incorrect choices are selected. Select all that apply. 1. Use triple-dye to cleanse the umbilical cord at home. 2. Fold the diaper down to prevent covering the cord stump. 3. Keep the umbilical stump clean and dry to avoid infection. 4. Observe for signs of infection such as foul smell, redness, and drainage. 5. Begin tub baths to help cleanse the cord stump at home.

Answer: 2, 3, 4 Explanation: 2. Folding the diaper down to prevent coverage of the cord stump can prevent contamination of the area and promote drying. 3. Keeping the umbilical stump clean and dry can reduce the risk of infection. 4. It is the nurse's responsibility to instruct parents in caring for the cord and observing for signs and symptoms of infection after discharge, such as foul smell, redness and greenish yellow drainage, localized heat and tenderness, or bright red bleeding or if the area remains unhealed 2 to 3 days after the cord has sloughed off.

Prior to conducting the initial assessment of a newborn, the nurse reviews the mother's prenatal record and the delivery record to obtain information concerning possible risk factors for the infant and to anticipate the impact of these factors on the infant's ability to successfully transition to the extrauterine environment. Which information is pertinent to this assessment? Note: Credit will be given only if all correct choices and no incorrect choices are selected. Select all that apply. 1. Drug or alcohol use by the father 2. Infectious disease screening results 3. Maternal history of gestational diabetes 4. Prolonged rupture of the membranes 5. Maternal use of prenatal vitamins

Answer: 2, 3, 4 Explanation: 2. Infectious disease screening results help to determine if the infant is also at risk of obtaining any infectious diseases. 3. Gestational diabetes is a risk factor for the newborn. 4. Prolonged rupture of the membranes is a possible risk factor for the infant.

The nurse is planning care for a preterm newborn. Which nursing diagnosis has the highest priority? 1. Tissue Integrity, Impaired 2. Infection, Risk for 3. Gas Exchange, Impaired 4. Family Processes, Dysfunctional

Answer: 3 Explanation: 3. Gas Exchange, Impaired is related to immature pulmonary vasculature and inadequate surfactant production and has the highest priority.

When caring for the menopausal woman, nurses need to be empathetic in approaching which of the following areas? Note: Credit will be given only if all correct and no incorrect choices are selected. Select all that apply. 1. Administering medications 2. Health teaching 3. Providing physical care 4. Counseling 5. Encouraging hormone therapy

Answer: 2, 3, 4 Explanation: 2. The nurse needs to use an empathetic approach in counseling, health teaching, and providing physical care. 3. The nurse needs to use an empathetic approach in counseling, health teaching, and providing physical care. 4. The nurse needs to use an empathetic approach in counseling, health teaching, and providing physical care.

In providing community education about hepatitis, the nurse includes information on the chronic forms of hepatitis. A form of hepatitis that becomes chronic is which of the following? Note: Credit will be given only if all correct and no incorrect choices are selected. Select all that apply. 1. Hepatitis A 2. Hepatitis B 3. Hepatitis C 4. Hepatitis D 5. Hepatitis E

Answer: 2, 3, 4 Explanation: 2. Unlike hepatitis A infection, hepatitis B infection is chronic. 3. Unlike hepatitis A infection, hepatitis C infection is chronic. 4. Unlike hepatitis A infection, hepatitis D infection is chronic.

The nurse educator teaching reproductive anatomy wants to make sure the students understand what stabilizes the uterus. Which statements about the individual ligaments would the nurse include? Note: Credit will be given only if all correct choices and no incorrect choices are selected. Select all that apply. 1. The infundibulopelvic ligaments suspend and support the uterus. 2. The broad ligament keeps the uterus centrally placed. 3. The uterosacral ligaments sweep back around the rectum and insert on the sides of the first and second sacral vertebrae. 4. The ovarian ligaments anchor the ovary to the uterus. 5. The cardinal ligaments prevent uterine prolapse and support the upper vagina.

Answer: 2, 3, 4, 5 Explanation: 2. The broad ligament keeps the uterus centrally placed. 3. The uterosacral ligaments sweep back around the rectum and insert on the sides of the first and second sacral vertebrae. 4. The ovarian ligaments anchor the ovary to the uterus. 5. The cardinal ligaments prevent uterine prolapse and support the upper vagina.

A client in the clinic asks the nurse about available contraceptives. Before responding, the nurse must assess which of the following factors? Note: Credit will be given only if all correct and no incorrect choices are selected. Select all that apply. 1. When menarche occurred 2. How frequently the client has intercourse 3. Whether the client has a history of thrombophlebitis 4. What the client's partner prefers 5. Whether the client is in a monogamous relationship

Answer: 2, 3, 5 Explanation: 2. A factor to consider when choosing a method of contraception is how frequently the client has intercourse. 3. A factor to consider when choosing a method of contraception is whether the client has a history of thrombophlebitis or any other condition that would contraindicate its use. 5. A factor to consider when choosing a method of contraception is whether the client is in a monogamous relationship.

The nurse working with pediatric clients knows that the primary hormone secretions that induce puberty include which of the following? Note: Credit will be given only if all correct and no incorrect choices are selected. Select all that apply. 1. Thyroid hormone 2. Follicle-stimulating hormone 3. Leuteinizing hormone 4. Adrenocorticotropic hormone 5. Gonadotropin-releasing hormones

Answer: 2, 3, 5 Explanation: 2. Follicle-stimulating hormone (FSH) is part of the process that induces puberty. 3. The luteinizing hormone (LH) is a part of the process that induces puberty. 5. The central nervous system releases a neurotransmitter that stimulates the hypothalamus to synthesize and release gonadotropin-releasing hormone (GnRH).

A NICU nurse plans care for a preterm newborn that will provide opportunities for development. Which interventions support development in a preterm newborn in a NICU? Note: Credit will be given only if all correct choices and no incorrect choices are selected. Select all that apply. 1. Schedule care throughout the day. 2. Silence alarms quickly. 3. Place a blanket over the top portion of the incubator. 4. Do not offer a pacifier. 5. Dim the lights.

Answer: 2, 3, 5 Explanation: 2. Noise levels can be lowered by replacing alarms with lights or silencing alarms quickly. 3. Dimmer switches should be used to shield the baby's eyes from bright lights with blankets over the top portion of the incubator. 5. Dimming the lights may encourage infants to open their eyes and be more responsive to their parents.

The nurse interviews a 28-year-old client with a new medical diagnosis of endometriosis. Which question asked by the nurse is appropriate? Note: Credit will be given only if all correct and no incorrect choices are selected. Select all that apply. 1. "Are you having hot flashes?" 2. "Are you experiencing pain during intercourse?" 3. "Is a vaginal discharge present?" 4. "Are you having pain during your period?" 5. "Have you noticed any skin rashes?"

Answer: 2, 4 Explanation: 2. The primary symptoms of endometriosis include dyspareunia. 4. The primary symptoms of endometriosis include dysmenorrhea.

Benefits of skin-to-skin care as a developmental intervention include which of the following? Note: Credit will be given only if all correct choices and no incorrect choices are selected. Select all that apply. 1. Routine discharge 2. Stabilization of vital signs 3. Increased periods of awake-alert state 4. Decline in the episodes of apnea and bradycardia 5. Increased growth parameters

Answer: 2, 4, 5 Explanation: 2. Stabilization of vital signs is a benefit of skin-to-skin care as a developmental intervention. 4. Decline in the episodes of apnea and bradycardia is a benefit of skin-to-skin care as a developmental intervention. 5. Increased growth parameters are a benefit of skin-to-skin care as a developmental intervention.

What are the three functions of cervical mucosa? Note: Credit will be given only if all correct choices and no incorrect choices are selected. Select all that apply. 1. Form the relatively fixed axis of the birth passage 2. Provide lubrication for the vaginal canal 3. Provide nourishment and protective maternal antibodies to infants 4. Provide an alkaline environment to shelter deposited sperm from the acidic vaginal secretions 5. Act as a bacteriostatic agent

Answer: 2, 4, 5 Explanation: 2. The cervical mucosa provides lubrication for the vaginal canal. 4. The cervical mucosa provides an alkaline environment to shelter deposited sperm from the acidic vaginal secretions. 5. The cervical mucosa acts as a bacteriostatic agent.

The nurse in the community clinic instructs a client starting on a combined oral contraceptive that she must immediately report which of the following symptoms? Note: Credit will be given only if all correct and no incorrect choices are selected. Select all that apply. 1. Only four withdrawal bleeds per year 2. Speech problems 3. Unscheduled bleeding 4. Blood pressure of 126/84 5. New lumps in her breast

Answer: 2, 5 Explanation: 2. The woman using combined oral contraceptives should contact her healthcare provider if she develops speech problems. 5. The woman using combined oral contraceptives should contact her healthcare provider if she develops a breast lump.

What is the term for when children alternate between two homes, spending varying amounts of time with each parent in a situation called co-parenting and usually involving joint custody? 1. Blended or reconstituted nuclear family 2. Extended kin network family 3. Binuclear family 4. Extended family

Answer: 3 Explanation: 3. A binuclear family is a postdivorce family in which the biologic children are members of two nuclear households, with parenting by both the father and the mother.

The nurse in the community should use a family assessment tool to obtain what type of information? 1. How long the family has lived at its current address 2. What other health insurance the family has had in the past 3. How the family meets its nutritional needs and obtains food 4. What eye color the family desires in its unborn child

Answer: 3 Explanation: 3. A family assessment is a collection of data about the family's type and structure, current level of functioning, support system, sociocultural background, environment, and needs.

The nurse is developing a teaching plan for a client undergoing a tubal ligation. What information should be included in the plan? 1. The surgical procedure is easily reversible. 2. Laparotomy is performed following a vaginal birth. 3. Minilaparotomy is performed in the postpartum period soon after a vaginal birth. 4. Tubal ligation can be done at any time the woman is either pregnant or not pregnant.

Answer: 3 Explanation: 3. A tubal ligation minilaparotomy is performed in the postpartum period soon after a vaginal birth.

The nurse is assessing a client who reports seeing an acupuncturist on a weekly basis to treat back pain. The nurse understands that acupuncture is an example of what? 1. A risky practice without evidence of efficacy 2. A folk remedy 3. A complementary therapy 4. An alternative therapy

Answer: 3 Explanation: 3. Acupuncture is a therapy that is used in conjunction with conventional medical treatment, and therefore is an example of a complementary therapy.

The client diagnosed with endometriosis asks the nurse whether there are any long-term health risks associated with this condition. The nurse should include which statement in the client teaching about endometriosis? 1. "There are no other health risks associated with endometriosis." 2. "Pain with intercourse rarely occurs as a long-term problem." 3. "You are at increased risk for ovarian and breast cancer." 4. "Most women with this condition develop severe migraines."

Answer: 3 Explanation: 3. An increased risk for cancer of the ovary and breast is associated with endometriosis.

The parents of a newborn male ask the nurse whether they should circumcise their son. What is the best response by the nurse? 1. "Circumcision should be undertaken to prevent problems in the future." 2. "Circumcision might decrease the child's risk of developing a urinary tract infection." 3. "Circumcision can sometimes cause complications. What questions do you have?" 4. "Circumcision is painful, and should be avoided unless you are Jewish."

Answer: 3 Explanation: 3. Asking this question allows the nurse to determine what the parents' concerns are, then address them specifically.

The nurse in a rural clinic is talking with some clients about biofeedback. The nurse explains to the clients that biofeedback is which of the following? 1. An alternative therapy 2. A state of great mental and physical relaxation in which one is susceptible to suggestion 3. A method used to help individuals learn to control their physiologic responses based on the concept that the mind controls the body 4. A complementary therapy in which one goes into a relaxed state and focuses on positive scenes

Answer: 3 Explanation: 3. Biofeedback is a method to learn control of physiological responses based on the concept that the mind controls the body.

When is breastfeeding contraindicated? 1. Infant has hypertension 2. Mother has a history of treated tuberculosis 3. Mother is HIV positive or has AIDS 4. Mother has a history of treated herpes

Answer: 3 Explanation: 3. Breastfeeding is contraindicated when the mother is HIV positive or has AIDS and is counseled against breastfeeding.

The nurse is planning a community education program on the role of complementary and alternative therapies during pregnancy. Which statement about alternative and complementary therapies should the nurse include? 1. "They bring about cures for illnesses and diseases." 2. "They are invasive but effective for achieving health." 3. "They emphasize prevention and wellness." 4. "They prevent pregnancy complications."

Answer: 3 Explanation: 3. Complementary and alternative therapies have many benefits during pregnancy. They emphasize prevention and wellness, aiming for holistic health rather than cure or treatment.

The nurse is planning an educational session about contraception. What will the nurse emphasize as being the most significant factor in determining the effectiveness of a specific method of contraception to avoid pregnancy? 1. Reliability 2. Ease of use 3. Consistency of use 4. Cost

Answer: 3 Explanation: 3. Consistency of use is the most important factor for determining the effectiveness of a specific type of contraception.

The nurse is caring for a client diagnosed with endometriosis. Which statement by the client would require a need for perhaps another treatment option? 1. "I am having many hot flashes since I had the Lupron injection." 2. "The pain I experience with intercourse is becoming more severe." 3. "I have vaginal dryness, reduced libido, and my clitoris has become larger since taking danazol. Is this normal?" 4. "I've noticed I have not had my period on a regular basis since being on the GnRH analogs."

Answer: 3 Explanation: 3. Danazol is a testosterone derivative that suppresses ovulation and causes amenorrhea. It is intended for short-term therapy. Because of adverse effects, many clinicians have moved away from danazol to other treatment options.

The nurse educator is lecturing on the changes that take place during puberty. What is a change that girls experience? 1. Elongation of the hips 2. Deepening of the voice 3. Broadening of the hips and budding of breasts 4. Preparation of the uterus for pregnancy

Answer: 3 Explanation: 3. During puberty, girls' hips start to broaden and their breasts begin to form.

The nurse is preparing a presentation on the menstrual cycle for a group of high school students. Which statement should the nurse include in this presentation? 1. "The menstrual cycle has five distinct phases that occur during the month." 2. "One hormone controls the phases of the menstrual cycle." 3. "The secretory phase occurs when a woman is most fertile." 4. "Menstrual cycle phases vary in order from one woman to another."

Answer: 3 Explanation: 3. During the secretory phase, the vascularity of the entire uterus increases greatly, providing a nourishing bed for implantation.

The nurse is taking a history on a new client in the clinic. She determines from talking with the client that she is recently divorced, is dating, and has had sex with various men. The nurse would be concerned about and would provide some education on what issues? 1. The ethics of dating and having sex with more than one man 2. The client having some kind of permanent birth control done, so she does not become pregnant 3. Education about sexual activity and sexually transmitted infections 4. Referral to a psychologist or counselor for follow-up on the multiple dating

Answer: 3 Explanation: 3. Education about sexual activity and sexually transmitted infections is correct, since it has been determined that the client is having sex with multiple partners.

A 49-year-old client comes to the clinic with complaints of severe perimenopausal symptoms including hot flashes, night sweats, urinary urgency, and vaginal dryness. The physician has prescribed a combination hormone replacement therapy of estrogen and progestin. When the client asks the nurse why she must take both hormones, what is the nurse's best reply? 1. "Hot flashes respond better when replacement includes both hormones." 2. "You are having very severe symptoms, so you need more hormones replaced." 3. "There is an increased risk of tissue abnormality inside the uterus if only one is given." 4. "Your blood pressure can become elevated if only one hormone is used."

Answer: 3 Explanation: 3. Estrogen alone, in a woman with a uterus (unopposed estrogen), increases the risk of endometrial (the lining of the uterus) cancer by eightfold and, therefore, is never given without progesterone in these women.

The student nurse encounters a 15-year-old girl who reports that she has no pubic or axillary hair and has not yet experienced growth of her breasts. The student asks the nurse about the physiology of this occurrence. The nurse explains that the client probably lacks which hormone? 1. Testosterone 2. Progesterone 3. Estrogen 4. Prolactin

Answer: 3 Explanation: 3. Estrogens influence the development of secondary sex characteristics.

The nurse is teaching a group of women about menopause at a community clinic. The nurse tells them that the best indicator of menopause is which of the following symptoms? 1. No menses for 8 consecutive months 2. Hot flashes and night sweats 3. FSH levels rise and ovarian follicles cease to produce estrogen 4. Diagnosed with osteoporosis 4 months ago

Answer: 3 Explanation: 3. Examining FSH and estrogen levels is a very accurate indication of menopause.

The nurse is conducting health screening at a community clinic. The client has asked whether there are any risks with body piercing and tattooing, or whether these activities would impact sexual activity. How should the nurse respond? 1. "You should avoid piercing your genitalia and your nipples." 2. "There are no problems that occur with either body piercing or tattooing." 3. "Both piercing and tattooing carry risks of infection, including hepatitis." 4. "The benefit of body art outweighs

Answer: 3 Explanation: 3. For tattooing and body piercing, risks include infections such as HIV and hepatitis B and C because of the use of inadequately sterilized equipment, as well as allergic reactions, local swelling and burns, granulomas, and keloid formation.

New parents decide not to have their newborn circumcised. What should the nurse teach regarding care for the uncircumcised infant? 1. The foreskin will be retractable at 2 months. 2. Retract the foreskin and clean thoroughly. 3. Avoid retracting the foreskin. 4. Use soap and Betadine to cleanse the penis daily.

Answer: 3 Explanation: 3. Foreskin will retract normally over time and may take 3 to 5 years.

A nurse is examining a client with a potential of polycystic ovarian syndrome. The nurse knows that which diagnostic is the most important indicator to support a diagnosis of polycystic ovarian syndrome? 1. Total estrogen level 2. Waist to chest ratio 3. Hyperandrogenism 4. Hypoinsulinemia

Answer: 3 Explanation: 3. Hyperandrogenism, as women with PCOS consistently have elevated serum androgen levels. These elevated androgen levels often lead to clinical manifestations such as hirsutism (excessive hair growth), acne, deepening voice, and increased muscle mass.

The nurse teaches the parents of an infant who recently was circumcised to observe for bleeding. What should the parents be taught to do if bleeding does occur? 1. Wrap the diaper tightly. 2. Clean with warm water with each diaper change. 3. Apply gentle pressure to the site with gauze. 4. Apply a new petroleum ointment gauze dressing.

Answer: 3 Explanation: 3. If bleeding does occur, apply light pressure with a sterile gauze pad to stop the bleeding within a short time. If this is not effective, contact the physician immediately or take the newborn to the healthcare provider.

A woman is experiencing mittelschmerz and increased vaginal discharge. Her temperature has increased by 0.6°C (1.0°F) over the past 36 hours. This most likely indicates what? 1. Menstruation is about to begin. 2. Ovulation will occur soon. 3. Ovulation has occurred. 4. She is pregnant, and will not menstruate.

Answer: 3 Explanation: 3. In some women, ovulation is accompanied by mid-cycle pain, known as mittelschmerz. This pain may be caused by a thick tunica albuginea or by a local peritoneal reaction to the expelling of the follicular contents and body temperature increases about 0.3°C to 0.6°C (0.5°F to 1°F) 24 to 48 hours after the time of ovulation.

During discharge planning for a drug-dependent newborn, the nurse explains to the mother how to do which of the following? 1. Place the newborn in a prone position. 2. Limit feedings to three a day to decrease diarrhea. 3. Place the infant supine and operate a home apnea-monitoring system. 4. Wean the newborn off the pacifier.

Answer: 3 Explanation: 3. Infants with neonatal abstinence syndrome are at a significantly higher risk for sudden infant death syndrome (SIDS) when the mother used heroin, cocaine, or opiates. The infant should sleep in a supine position, and home apnea monitoring should be implemented.

The nurse is assessing a drug-dependent newborn. Which symptom would require further assessment by the nurse? 1. Occasional watery stools 2. Spitting up after feeding 3. Jitteriness and irritability 4. Nasal stuffiness

Answer: 3 Explanation: 3. Jitteriness and irritability can be an indicator of drug withdrawal.

The client reports using an alternative therapy that involves the manipulation of soft tissues. This therapy has reduced the client's stress, diminished pain, and increased circulation. Which therapy has this client most likely received? 1. Guided imagery 2. Homeopathy 3. Massage therapy 4. Reflexology

Answer: 3 Explanation: 3. Massage therapy involves the manipulation of soft tissues.

The nurse assesses the gestational age of a newborn and informs the parents that the newborn is premature. Which of the following assessment findings is not congruent with prematurity? 1. Cry is weak and feeble 2. Clitoris and labia minora are prominent 3. Strong sucking reflex 4. Lanugo is plentiful

Answer: 3 Explanation: 3. Poor suck, gag, and swallow reflexes are characteristic of a preterm newborn.

A 19-year-old woman comes to the gynecologist's office. When the nurse asks the reason for this visit, the client explains that she has never had a menstrual period, and that she is concerned there might be something wrong. What is the diagnosis that the physician is most likely to make based on this information? 1. Primary dysmenorrhea 2. Secondary infertility 3. Primary amenorrhea 4. Secondary amenorrhea

Answer: 3 Explanation: 3. Primary amenorrhea is the term for the condition when menses have never occurred.

The nurse is preparing educational materials at a family planning clinic. The client who is an appropriate candidate for using emergency contraception would be one who reports which of the following? 1. Forgetting to start her pill pack yesterday 2. Unprotected intercourse during her menses 3. That a condom broke yesterday in the middle of her cycle 4. Increased dysmenorrhea since IUC insertion

Answer: 3 Explanation: 3. Research indicates that oral hormonal EC taken as soon as possible within 72 hours, but up to 5 days, can reduce the risk of pregnancy after a single act of unprotected intercourse by at least 74%.

The client who was raped is extremely upset when a pregnancy test confirms that she is pregnant, and requests information regarding pregnancy termination. Which statement is best for the nurse to include? 1. "Abortion is morally wrong, and should not be undertaken." 2. "Hypertension is a risk with any abortion." 3. "Surgical abortion in the first trimester is technically easier and safer than abortion in the second trimester." 4. "The most accurate method to determine gestational age are the results of a pregnancy test."

Answer: 3 Explanation: 3. Second-trimester abortion (greater than 13 weeks' gestation up to 24 weeks or as per state law) may be done medically or surgically.

A nurse teaching a sex education class is asked by a male student, "What exactly happens when my body gets aroused?" The nurse's reply includes which statement? 1. "The vas deferens thickens and expands." 2. "The sympathetic nerves of the penis are stimulated." 3. "The penis elongates, thickens, and stiffens." 4. "The prepuce of the penis elongates."

Answer: 3 Explanation: 3. Sexual stimulation causes the penis to elongate, thicken, and stiffen, a process called erection. The penis becomes erect when its blood vessels become engorged.

A client from Mexico has just delivered a son, and the nurse offers to assist in putting the baby to breast. Although the client indicated before the birth that she wanted to breastfeed, she is very hesitant, and says she would like to bottle-feed for the first few days. After talking to her, the nurse understands that her primary reason for wanting to delay breastfeeding is based on what cultural belief? 1. Breast milk causes skin rashes. 2. It is harmful to breastfeed immediately. 3. Colostrum is bad for the baby. 4. Thin milk causes diarrhea.

Answer: 3 Explanation: 3. Some Hispanics may delay breastfeeding because they believe colostrum is "bad."

A mother states that her breasts leak between feedings. Which of the following can contribute to the letdown reflex in breastfeeding mothers? 1. Pain with breastfeeding 2. Number of hours passed since last feeding 3. The newborn's cry 4. Maternal fluid intake

Answer: 3 Explanation: 3. Some women will leak milk when their breasts are full and it is nearly time to breastfeed again or whenever they experience letdown, which can be triggered by hearing, seeing, or even thinking of their baby.

Which of the following diagnostic tests would the nurse question when ordered for a client diagnosed with pelvic inflammatory disease (PID)? 1. CBC (complete blood count) with differential 2. Venereal Disease Research Laboratory (VDRL) 3. Throat culture for Streptococcus A 4. RPR (Rapid Plasma Reagin)

Answer: 3 Explanation: 3. Streptococcus of the throat is not associated with PID.

The nurse has instructed a new mother on quieting activities for her newborn. The nurse knows that the mother understands when she overhears the mother telling the father to do what? 1. Hold the newborn in an upright position. 2. Massage the hands and feet. 3. Swaddle the newborn in a blanket. 4. Make eye contact while talking to the newborn.

Answer: 3 Explanation: 3. Swaddling or bundling the baby increases a sense of security and is a quieting activity.

Which of the following systems provides a uniform format and classification of terminology based on current understanding of cervical disease? 1. Levonorgestrel intrauterine 2. PALM-COEIN 3. Bethesda 4. BSE

Answer: 3 Explanation: 3. The Bethesda System for classifying Pap smears is a standardized method of reporting cytologic Pap smear findings and is the most widely used method in the United States.

The nurse is instructing a new mother on circumcision care with a Plastibell. The nurse knows the mother understands when she states that the Plastibell should fall off within how long? 1. 2 days 2. 10 days 3. 8 days 4. 14 days

Answer: 3 Explanation: 3. The Plastibell should fall off within 8 days. If it remains on after 8 days, the parents should consult with the newborn's physician.

A client asks the nurse, "Can you explain to us how to use the basal body temperature method to detect ovulation and prevent pregnancy?" What response by the nurse is best? 1. "Take your temperature every evening at the same time and keep a record for a period of several weeks. A noticeable drop in temperature indicates that ovulation has occurred." 2. "Take your temperature every day at the same time and keep a record of the findings. A noticeable rise in temperature indicates ovulation." 3. "Take your temperature each day, immediately upon awakening, and keep a record of each finding. A noticeable rise in temperature indicates that ovulation is about to occur." 4. "This is an unscientific and unproven method of determining ovulation, and is not recognized as a means of birth control."

Answer: 3 Explanation: 3. The basal body temperature method is used to detect ovulation by an increase in the basal temperature during the menstrual cycle. It requires that the woman take her temperature every morning upon awakening (before any activity) and record the findings on a temperature graph, and is based on the fact that the temperature almost always rises and remains elevated after ovulation because of the production of progesterone, a thermogenic (heat-producing) hormone.

A client has been diagnosed with bacterial vaginosis. The nurse obtains a sexual history from the client, including contraceptive measures, number of sexual partners, and frequency of intercourse. What is the rationale for the questions? 1. Clients can infect their sexual partners. 2. The nurse is required by law to ask the questions. 3. Clients with bacterial vaginosis can become infected with HIV and other sexually transmitted diseases more easily. 4. The laboratory needs a full client history in order to know for which organisms and antibiotic sensitivities it should test.

Answer: 3 Explanation: 3. The change in normal flora increases the woman's susceptibility to other organisms, making the client more vulnerable to sexually transmitted diseases, including HIV.

The nurse is providing instructions for a woman who has just had intrauterine contraception inserted. What is essential for the nurse to teach the client? 1. To palpate her lower abdomen each month to check the patency of the device 2. To remain on bed rest for 24 hours after insertion of the device 3. To report any pain during intercourse to the physician 4. To insert spermicidal jelly within 4 hours of every sexual encounter

Answer: 3 Explanation: 3. The client should report any pain during intercourse to the physician. This can be due to an infection.

When the nurse is teaching a woman about the use of a diaphragm, it is important to instruct her that the diaphragm should be rechecked for correct size how often? 1. Every five years routinely 2. When weight gain or loss beyond five pounds has occurred 3. After each birth 4. Only after significant weight loss

Answer: 3 Explanation: 3. The diaphragm should be rechecked for correct size after each childbirth and whenever a woman has gained or lost 10 pounds or more.

In caring for pregnant clients, the nurse realizes that information on conventional, complementary, and alternative medicine is best obtained by which of the following means? 1. Obtained at the medical office if the physician feels it is appropriate 2. Obtained from family and friends who have already experienced a situation 3. Readily obtainable on the Internet 4. Passed on by word of mouth

Answer: 3 Explanation: 3. The dramatic increase in complementary and alternative therapies that began in the final decade of the 20th century is partially due to the advent of the Internet.

A school nurse teaching a health class to adolescent boys explains that spermatozoa become motile and fertile during the 2-10 days they are stored in which part of the male body? 1. Prostate gland 2. Vas deferens 3. Epididymis 4. Urethra

Answer: 3 Explanation: 3. The epididymis provides a reservoir where spermatozoa can survive for a long period and the spermatozoa usually remain in the epididymis for 2 to 10 days.

The nurse in the clinic instructs a client using the natural method of contraception to begin counting the first day of her cycle as which day? 1. The day her menstrual period ceases 2. The first day after her menstrual period ceases 3. The first day of her menstrual period 4. The day of ovulation

Answer: 3 Explanation: 3. The first day of menstruation is the first day of the cycle.

The nurse is caring for an infant born at 37 weeks that weighs 1750 g (3 pounds 10 ounces). The head circumference and length are in the 25th percentile. What statement would the nurse expect to find in the chart? 1. Preterm appropriate for gestational age, symmetrical IUGR 2. Term small for gestational age, symmetrical IUGR 3. Preterm small for gestational age, asymmetrical IUGR 4. Preterm appropriate for gestational age, asymmetrical IUGR

Answer: 3 Explanation: 3. The infant is preterm at 37 weeks. Because the weight is below the 10th percentile, the infant is small for gestational age. Head circumference and length between the 10th and 90th percentiles indicate asymmetrical IUGR.

When a breastfeeding mother complains that her breasts are leaking milk, the nurse can offer which effective intervention? 1. Decrease the number of minutes the newborn is at the breast per feeding. 2. Decrease the mother's fluid intake. 3. Place absorbent pads in the bra. 4. Administer oxytocin.

Answer: 3 Explanation: 3. The mother can wear nursing pads inside her bra with instructions to change wet pads frequently.

The nurse walks in to find the client crying after the physician informed her of her diagnosis of human papilloma virus (HPV). Which statement by the nurse conveys an attitude of acceptance toward the client with a sexually transmitted infection? 1. "Don't worry about it. In a few weeks, with treatment, the lesions will disappear." 2. "You seem upset. I'll get the doctor." 3. "You seem upset. Can I help answer any questions?" 4. "I think you need to see a therapist."

Answer: 3 Explanation: 3. The nurse's attitude of acceptance and matter-of-factness conveys to the client that she is still an acceptable person who happens to have an infection.

The nurse educator describes the uterus and ovaries as being held in place in the pelvic cavity by what structures? 1. Muscles 2. Tendons 3. Ligaments 4. Peritoneum

Answer: 3 Explanation: 3. The ovaries and uterus are held in place in the pelvic cavity by a number of ligaments.

A client is at a physician's office following the end of treatment for breast cancer. The client is not scheduled to receive any more radiation or chemotherapy at this time. With treatment behind her, the client can now look again to the future. Which phase of psychological adjustment is this client experiencing? 1. Shock 2. Reaction 3. Recovery 4. Reorientation

Answer: 3 Explanation: 3. The recovery phase begins during convalescence following the completion of medical treatment.

A prenatal client asks the nurse how the baby can possibly come out through her vagina, because a vagina is not nearly as big as a baby. How does the nurse best answer this client's question? 1. "The vagina usually tears as it stretches during childbirth." 2. "The vagina is designed to allow a baby come through." 3. "The vagina changes due to pregnancy allow the vagina to stretch more." 4. "The vagina dilates and effaces in labor so the baby can get out."

Answer: 3 Explanation: 3. The walls of the vagina are covered with ridges, or rugae, crisscrossing each other. These rugae allow the vaginal tissues to stretch enough for the fetus to pass through during childbirth.

A woman comes into the emergency department stating she was raped the day before, and requests emergency contraception. She states that she is in the middle of her cycle. The nurse knows which of the following about the client? 1. This client has waited too long before coming to the emergency department. 2. The client will have to wait 10 days before she can have emergency contraception. 3. The client can receive emergency contraception up to 72 hours to 5 days after the intercourse. 4. The client will have to wait and see whether she misses a period.

Answer: 3 Explanation: 3. This client is within the 72-hour to 5-day timeframe, and is able to receive the medicine.

The nurse educator is talking with a group of students doing their gynecological rotation. The nurse describes a number of symptoms that include anovulation, reduced fertility, either decreased or increased menstrual flow, and menstrual cycle irregularities. The nurse is describing what condition? 1. Intermenstrual bleeding 2. Hypermenorrhea 3. Menopause 4. Primary amenorrhea

Answer: 3 Explanation: 3. This is the correct answer. Anovulation, reduced fertility, either decreased or increased menstrual flow, and menstrual cycle irregularities are some of the symptoms of menopause.

The nurse is instructing a client in the use of combined oral contraceptives. The nurse knows that additional teaching is required when the client makes which statement about the risks of taking combined oral contraceptives? 1. "They will lower my risk for breast cancer." 2. "They will increase my risk for cervical cancer." 3. "They will lower my risk for ovarian cancer." 4. "They will increase my risk for uterine cancer."

Answer: 3 Explanation: 3. This is true. There is a reduction in the incidence of ovarian cancer.

A client who wants to use the vaginal sponge method of contraception shows that she understands the appropriate usage when she makes which statement? 1. "I need to use a lubricant prior to insertion." 2. "I need to add spermicidal cream prior to intercourse." 3. "I need to moisten it with water prior to use." 4. "I need to leave it in no longer than 6 hours."

Answer: 3 Explanation: 3. To activate the spermicide in the vaginal sponge, it must be moistened thoroughly with water.

The transcultural nursing theory was developed in 1961 by Dr. Madeleine Leininger. Its foundation is in which of the following? 1. The framework categorizes a family's progression over time 2. The family life cycle of a traditional nuclear family 3. Anthropology and nursing 4. Holistic health beliefs

Answer: 3 Explanation: 3. Transcultural nursing theory is rooted in the caring that embraces the beliefs and practices of individuals or groups of similar or different cultures.

The client has been found to have vitiligo on her vulva. Which client statement indicates that the client requires additional teaching on this condition? 1. "This can occur in other places on my body." 2. "Vitiligo is only a decrease of pigmentation." 3. "Other serious health conditions are common with vitiligo." 4. "There usually are no other symptoms of this condition."

Answer: 3 Explanation: 3. Vitiligo is an absence of melanin, which results in white patches that are especially noticeable on dark-skinned individuals. It can occur in multiple areas on the body, and has no associated symptoms or health conditions.

) The nurse is teaching a client who has been diagnosed with vulvitis. Which statement by the client indicates that the nurse's instruction has not been effective? 1. "I should stop having sexual intercourse." 2. "Non-deodorized tampons could make this condition recur." 3. "Wearing pantyhose daily will improve the problem." 4. "A different brand of soap might eliminate the irritation."

Answer: 3 Explanation: 3. Vulvitis is inflammation of the vulva. Tight clothing, especially if made of synthetic fibers, can predispose women to the condition. Pantyhose should not be worn.

The nurse knows that in some cases, breastfeeding is not advisable. Which mother should be counseled against breastfeeding? 1. A mother with a poorly balanced diet 2. A mother who is overweight 3. A mother who is HIV positive 4. A mother who has twins

Answer: 3 Explanation: 3. Women with HIV or AIDS are counseled against breastfeeding.

Women with pyelonephritis during pregnancy are at significantly increased risk for which condition? 1. Foul-smelling discharge 2. Ectopic pregnancy 3. Preterm labor 4. A colicky large intestine

Answer: 3 Explanation: 3. Women with pyelonephritis during pregnancy are at significantly increased risk of preterm labor, preterm birth, development of adult respiratory distress syndrome, and septicemia.

The nurse is providing discharge instructions to a client with a diagnosis of vulvovaginal candidiasis (VVC), and knows the client understands when she makes which of the following statements? 1. "I need to apply the miconazole for 10 days." 2. "I need to douche daily." 3. "I need to add yogurt to my diet." 4. "I need to wear nylon panties."

Answer: 3 Explanation: 3. Yogurt helps reestablish normal vaginal flora.

A nurse is providing a client with instructions regarding breast self-examination (BSE). Which of the following statements by the client would indicate that the teaching has been successful? Note: Credit will be given only if all correct choices and no incorrect choices are selected. Select all that apply. 1. "I should perform BSE 1 week prior to the start of my period." 2. "When I reach menopause, I will perform BSE every 2 months." 3. "Knowing the density of my breast tissue is important." 4. "I should inspect my breasts while standing with my arms down at my sides." 5. "I should inspect my breasts while in a supine position with my arms at my sides."

Answer: 3, 4 Explanation: 3. The effectiveness of BSE is determined by the woman's ability to perform the procedure correctly, by her knowledge of her own breast tissue, and by the density of her breast tissue. 4. The breasts should be inspected while standing with arms at sides.

The nurse is working with parents who have just experienced the birth of their first child at 34 weeks. Which statements by the parents indicate that additional teaching is needed? Note: Credit will be given only if all correct choices and no incorrect choices are selected. Select all that apply. 1. "Our baby will be in an incubator to keep him warm." 2. "Breathing might be harder for our baby because he is early." 3. "The growth of our baby will be faster than if he were term." 4. "Tube feedings will be required because his stomach is small." 5. "Because he came early, he will not produce urine for 2 days."

Answer: 3, 4, 5 Explanation: 3. Preterm infants grow more slowly than do term infants because of difficulty in meeting high caloric and fluid needs for growth due to small gastric capacity. 4. Although tube feedings might be required, it would be because preterm babies have a marked danger of aspiration and its associated complications due to the infant's poorly developed gag reflex, incompetent esophageal cardiac sphincter, and inadequate suck/swallow/breathe reflex. 5. Although preterm babies have diminished kidney function due to incomplete development of the glomeruli, they can produce urine. Preterm infants usually have some urine output during the first 24 hours of life.

What should the healthcare provider consider when prescribing a medication to a woman who is breastfeeding? Note: Credit will be given only if all correct choices and no incorrect choices are selected. Select all that apply. 1. Drug's potential effect on hormone production 2. Amount of drug excreted into the mother's blood 3. Drug's potential adverse effects to the infant 4. Infant's age and health 5. Mother's need for the medication

Answer: 3, 4, 5 Explanation: 3. The healthcare provider should consider the drug's potential adverse effects to the infant. 4. The healthcare provider should consider the infant's age and health. 5. The healthcare provider should consider the mother's need for the medication

A 7 pound 14 ounce girl was born to an insulin-dependent type II diabetic mother 2 hours ago. The infant's blood sugar is 47 mg/dL. What is the best nursing action? 1. To recheck the blood sugar in 6 hours 2. To begin an IV of 10% dextrose 3. To feed the baby 1 ounce of formula 4. To document the findings in the chart

Answer: 4 Explanation: 4. A blood sugar level of 47 mg/dL is a normal finding; documentation is an appropriate action.

The nurse is discharging a 15-year-old first-time mother. Which statement should the nurse include in the discharge teaching? 1. "Call your pediatrician if the baby's temperature is below 98.6°F axillary." 2. "Your baby's stools will change to a greenish color when your milk comes in." 3. "You can wipe away any eye drainage that might form." 4. "Your infant should wet a diaper at least 6 times per day."

Answer: 4 Explanation: 4. A minimum of 6 to 10 wet diapers per day indicates adequate fluid intake.

A client asks the nurse about treatment for human papilloma viral warts. The nurse's response should be based on what knowledge? 1. An antiviral injection cures approximately 50% of all cases. 2. Aggressive treatment is required to cure warts. 3. Warts often spread when an attempt is made to remove them surgically. 4. No single treatment is best for all types of warts or for all clients.

Answer: 4 Explanation: 4. All atypical, pigmented, and persistent warts should be biopsied and treatment instituted promptly.

The nurse teaching a client describes the effect of a vasectomy on fertilization by saying a man who has had a vasectomy becomes functionally sterile because of which of the following? 1. "Sperm are no longer being produced." 2. "Sperm are no longer motile and fertile." 3. "Sperm sit in the testes where they are formed." 4. "Sperm cannot reach the outside of the body."

Answer: 4 Explanation: 4. Although sperm continue to be produced for the next several years, they can no longer reach the outside of the body.

The nurse is caring for a new breastfeeding mother who is from Pakistan. The nurse plans her care so that the newborn is offered the breast on which of the following? 1. Day of birth 2. First day after birth 3. Second day after birth 4. Third to fourth day after birth

Answer: 4 Explanation: 4. Among some traditional cultures around the world, it is believed that colostrum is "unclean" or even harmful to a newborn. Because of this ancient belief, mothers living the Middle East and parts of Asia even today discard their colostrum or wait 2 to 4 days to begin breastfeeding, when their "true milk" arrives. This mother would begin breastfeeding the third or fourth day after the birth.

The client is undergoing lab work and ultrasound for a possible diagnosis of polycystic ovarian syndrome (PCOS). Which problem does the nurse expect to find in the client's history? 1. Multiple first-trimester fetal losses 2. Dyspareunia 3. Vulvitis 4. Oligomenorrhea

Answer: 4 Explanation: 4. Irregular menses, ranging from total absence of periods (amenorrhea) to intermittent or infrequent periods (oligomenorrhea) are the hallmarks of PCOS.

The nurse is discussing parent-infant attachment with a prenatal class. Which statement indicates that teaching was successful? 1. "I should avoid looking directly into the baby's eyes to prevent frightening the baby." 2. "My baby will be very sleepy immediately after birth and should go to the nursery." 3. "Newborns cannot focus their eyes, so it doesn't matter how I hold my new baby." 4. "Giving the baby his first bath can really give me a chance to get to know him."

Answer: 4 Explanation: 4. Another situation that can facilitate attachment is the interactive bath. While bathing their newborn for the first time, parents attend closely to their baby's behavior and the nurse can observe and point out behaviors.

) The nurse works in a facility that cares for clients from a broad range of racial, ethnic, cultural, and religious backgrounds. Which statement should the nurse include in a presentation to recently hired nurses on the client population of the facility? 1. "Our clients come from a broad range of backgrounds, but we have a good interpreter service." 2. "Many of our clients come from backgrounds different from your own, but it doesn't cause problems for the nurses." 3. "Because most of the doctors are bilingual, we don't have to deal with the differences in cultural backgrounds of our clients." 4. "Understanding the common values and health practices of our diverse clients will facilitate better care and health outcomes."

Answer: 4 Explanation: 4. Because of the implications for care based on cultural background, it is important for nurses to understand the backgrounds of the client population that accesses the facility. Without cultural awareness, caregivers tend to project their own cultural responses onto foreign-born clients; clients from different socioeconomic, religious, or educational groups; or clients from different regions of the country.

The nurse educator is talking with the students in the clinical area about amenorrhea. She has discussed both primary and secondary amenorrhea and their possible causes and knows that her teaching has been successful when, upon being questioned, a student explains that amenorrhea can be caused by which of the following? 1. Malfunctioning of the pancreas and insulin usage 2. Lack of testosterone after the time for menses to start 3. Lack of vitamin D and calcium in the system 4. Dysfunction of the hypothalamus

Answer: 4 Explanation: 4. Causes for amenorrhea include dysfunction of the hypothalamus, pituitary, and/or anovulation.

The nurse is analyzing assessment findings on four newborns. Which finding might suggest a congenital heart defect? 1. Apical heart rate of 140 beats per minute 2. Respiratory rate of 40 3. Temperature of 36.5°C 4. Visible, blue discoloration of the skin

Answer: 4 Explanation: 4. Central cyanosis is defined as a visible, blue discoloration of the skin caused by decreased oxygen saturation levels and is a common manifestation of a cardiac defect.

The nurse is preparing a community presentation on family development. Which statement should the nurse include? 1. The youngest child determines the family's current stage. 2. A family does not experience overlapping of stages. 3. Family development ends when the youngest child leaves home. 4. The stages describe the family's progression over time.

Answer: 4 Explanation: 4. Family development stages describe the changes and adaptations that a family goes through over time as children are added to the family.

) A nurse is teaching a middle school health class on the different types of viral hepatitis. Which statement made by a student indicates the need for further teaching? 1. "Both hepatitis A and E are not chronic infections." 2. "Hepatitis A is characterized by symptoms of jaundice, anorexia, nausea, vomiting, malaise, and fever." 3. "Hepatitis B, C, and D have symptoms similar to those of hepatitis A, and can also include arthralgias, arthritis and skin eruptions or rash." 4. "Both hepatitis B and C have an incubation period of 45-160 days."

Answer: 4 Explanation: 4. Hepatitis B has an incubation period of 45-160 days, but hepatitis C has an incubation period of 14-180 days.

A 38-week newborn is found to be small for gestational age (SGA). Which nursing intervention should be included in the care of this newborn? 1. Monitor for feeding difficulties. 2. Assess for facial paralysis. 3. Monitor for signs of hyperglycemia. 4. Maintain a warm environment.

Answer: 4 Explanation: 4. Hypothermia is a common complication in the SGA newborn; therefore, the newborn's environment must remain warm, to decrease heat loss.

The nurse teaching the phases of the menstrual cycle should include the fact that the corpus luteum begins to degenerate, estrogen and progesterone levels fall, and extensive vascular changes occur in which phase? 1. Menstrual phase 2. Proliferative phase 3. Secretory phase 4. Ischemic phase

Answer: 4 Explanation: 4. In the ischemic phase, the corpus luteum begins to degenerate, and as a result, both estrogen and progesterone levels fall. Small blood vessels rupture, and the spiral arteries constrict and retract, causing a deficiency of blood in the endometrium, which becomes pale.

The nurse is caring for a newborn in the special care nursery. The infant has hydrocephalus, and is positioned in a prone position. The nurse is especially careful to cleanse all stool after bowel movements. This care is most appropriate for an infant born with which of the following? 1. Omphalocele 2. Gastroschisis 3. Diaphragmatic hernia 4. Myelomeningocele

Answer: 4 Explanation: 4. Myelomeningocele is a saclike cyst containing meninges, spinal cord, and nerve roots in thoracic and/or lumbar area. Meticulous cleaning of the buttocks and genitals helps prevent infection. The infant is positioned on abdomen or on side and restrain (to prevent pressure and trauma to sac). Hydrocephalus often is present.

The nurse assesses the newborn and notes the following behaviors: nasal flaring, facial grimacing, and excessive mucus. What is the nurse most concerned about? 1. Neonatal jaundice 2. Neonatal hypothermia 3. Neonatal hyperthermia 4. Respiratory distress

Answer: 4 Explanation: 4. Nasal flaring and facial grimacing are signs of respiratory distress.

Before the newborn and mother are discharged from the birthing unit, the nurse teaches the parents about newborn screening tests that includes which of the following? 1. Preeclampsia screening 2. Congenital kidney disease screening 3. Visual screening 4. Hearing screening

Answer: 4 Explanation: 4. Newborn screening tests include hearing screening tests.

The nurse is instructing the parents of a newborn about car seat safety. Which statement indicates that the parents need additional information? 1. "The baby should be in the back seat." 2. "Newborns must be in rear-facing car seats." 3. "We need instruction on how to use the car seat before installing it." 4. "We can bring the baby home from the hospital without a car seat as it is only a short drive home."

Answer: 4 Explanation: 4. Newborns must go home from the birthing unit in a car seat adapted to fit newborns.

The nurse is analyzing various teaching strategies for teaching new mothers about newborn care. To enhance learning, which teaching method should the nurse implement? 1. Select videos on various topics of newborn care. 2. Organize a class that includes first-time mothers only. 3. Have mothers return in 1 week, when they feel more rested. 4. Schedule time for one-to-one teaching in the mother's room.

Answer: 4 Explanation: 4. One-to-one teaching while the nurse is in the mother's room is shown to be the most effective educational model. Individual instruction is helpful to answer specific questions.

The nurse is planning home visits to the homes of new parents and their newborns. Which client should the nurse see first? 1. 3-day-old male who received hepatitis B vaccine prior to discharge 2. 4-day-old female whose parents are both hearing-impaired 3. 5-day-old male with light, sticky, yellow drainage on the circumcision site 4. 6-day-old female with greenish discharge from the umbilical cord site

Answer: 4 Explanation: 4. Oozing of greenish yellow material, or reddened areas around the cord is not an expected finding. This family should be seen first because the child is experiencing a complication.

The neonatal special care unit nurse is overseeing the care provided by a nurse new to the unit. Which action requires immediate intervention? 1. The new nurse holds the infant after giving a gavage feeding. 2. The new nurse provides skin-to-skin care. 3. The new nurse provides care when the baby is awake. 4. The new nurse gives the feeding with room-temperature formula.

Answer: 4 Explanation: 4. Preterm babies have little subcutaneous fat, and do not maintain their body temperature well. Formula should be warmed prior to feedings to help the baby maintain its temperature.

The nurse is caring for a premature infant in the NICU, and is going to attempt a bottle feeding with thawed breast milk. How long can thawed breast milk be stored in the refrigerator before the nurse must discard it? 1. 4 hours 2. 8 hours 3. 12 hours 4. 24 hours

Answer: 4 Explanation: 4. Previously frozen thawed breast milk is good in the refrigerator for 24 hours only.

The nurse is presenting a community education session on female hormones. Which statement from a participant indicates the need for further information? 1. "Estrogen is what causes females to look female." 2. "The presence of some hormones causes other to be secreted." 3. "Progesterone is present at the end of the menstrual cycle." 4. "Prostaglandin is responsible for achieving conception."

Answer: 4 Explanation: 4. Prostaglandin is not related to conception. Prostaglandin production increases during follicular maturation and has basic regulatory functions in cells.

The nurse is working with new parents who have recently immigrated to the United States. The nurse is not familiar with the family's cultural background. Which approach is most appropriate when discussing the newborn? 1. "You appear to be Muslim. Do you want your son circumcised?" 2. "Let me explain newborn care here in the United States." 3. "Your baby is a United States citizen. You must be very happy about that." 4. "Could you explain your preferences regarding childrearing?"

Answer: 4 Explanation: 4. The nurse must be sensitive to the cultural beliefs and values of the family and be aware of cultural variations in newborn care.

When teaching a culturally diverse group of childbearing families about hospital birthing options, the culturally competent nurse does which of the following? 1. Understands that the families have the same values as the nurse 2. Teaches the families how childbearing takes place in the United States 3. Insists that the clients answer questions instead of their husbands 4. Incorporates the specific beliefs of the cultural groups that are attending the class

Answer: 4 Explanation: 4. Providing culturally competent care involves recognizing the importance of the childbearing family's value system, acknowledging that differences occur among people, and respecting and responding to ethnic diversity in a way that leads to mutually desirable outcomes.

The labor and delivery nurse is caring for a laboring client who has asked for a priest to visit her during labor. The client's mother died during childbirth, and although there are no complications during her pregnancy, the client is fearful of her own death during labor. What is the best response by the nurse? 1. "Nothing is going to happen to you. We'll take very good care of you during your birth." 2. "Would you like to have an epidural so that you won't feel the pain of the contractions?" 3. "The priest won't be able to prevent complications, and might get in the way of your providers." 4. "Would you like me to contact

Answer: 4 Explanation: 4. Providing spiritually sensitive care involves determining the current spiritual and religious beliefs and practices that will affect the mother and baby and accommodating these practices where possible.

A client at 20 weeks' gestation has not decided on a feeding method for her infant. She asks the nurse for advice. The nurse presents information about the advantages and disadvantages of formula-feeding and breastfeeding. Which statements by the client indicate that the teaching was successful? 1. "Formula-feeding gives the baby protection from infections." 2. "Breast milk cannot be stored; it has to be thrown away after pumping." 3. "Breastfeeding is more expensive than formula-feeding." 4. "My baby will have a lower risk of food allergies if I breastfeed."

Answer: 4 Explanation: 4. Secretory IgA, an immunoglobulin present in colostrum and mature breast milk, has antiviral, antibacterial, and antigenic-inhibiting properties and plays a role in decreasing the permeability of the small intestine to help prevent large protein molecules from triggering an allergic response.

Duvall's eight stages in the family life cycle of a traditional nuclear family have been used as the foundation for contemporary models that describe the developmental processes and role expectations for different family types. Which of the following is an example of Stage IV of this family life cycle? 1. Families launching young adults (all children leave home) 2. Families with preschool-age children (oldest child is between 2.5 and 6 years of age) 3. Middle-aged parents (empty nest through retirement) 4. Families with schoolchildren (oldest child is between 6 and 13 years of age)

Answer: 4 Explanation: 4. Stage IV is families with schoolchildren (oldest child is between 6 and 13 years of age).

Parents have been told their child has fetal alcohol syndrome (FAS). Which statement by a parent indicates that additional teaching is required? 1. "Our baby's heart murmur is from this syndrome." 2. "He might be a fussy baby because of this." 3. "His face looks like it does due to this problem." 4. "Cuddling and rocking will help him stay calm."

Answer: 4 Explanation: 4. The FASD baby is most comfortable in a quiet, minimally stimulating environment.

The nurse teaching a class on the reproductive system is discussing what happens at puberty. Which statement does the nurse make? 1. Boys and girls go through puberty at the same time. 2. Most girls develop breasts and start their menses at about the same time. 3. The nocturnal emissions that adolescent boys have contain a large number of sperm. 4. The onset and progress of puberty varies widely from person to person.

Answer: 4 Explanation: 4. The age at onset and progress of puberty vary widely, physical changes overlap, and the sequence of events can vary from person to person.

The nurse educator is discussing abdominal and vaginal hysterectomy with the students. The nurse explains that the one main disadvantage of a vaginal hysterectomy is which of the following? 1. More blood loss with the surgery 2. Increased pain postoperatively 3. A longer recuperation period 4. Trauma to the bladder

Answer: 4 Explanation: 4. The chance of doing some trauma to the bladder is greater with the vaginal hysterectomy.

What is the function of the scrotum? 1. Produce testosterone, the primary male sex hormone 2. Deposit sperm in the female vagina during sexual intercourse so that fertilization of the ovum can occur 3. Provide a reservoir where spermatozoa can survive for a long period 4. Protect the testes and the sperm by maintaining a temperature lower than that of the body

Answer: 4 Explanation: 4. The function of the scrotum is to protect the testes and the sperm by maintaining a temperature lower than that of the body.

A nonpregnant client is diagnosed with bacterial vaginosis (BV). What does the nurse expect to administer? 1. Penicillin G 2 million units IM one time 2. Zithromax 1 mg p.o. b.i.d. for 2 weeks 3. Doxycycline 100 mg p.o. b.i.d. for a week 4. Metronidazole 500 mg p.o. b.i.d. for a week

Answer: 4 Explanation: 4. The nonpregnant woman who is diagnosed with bacterial vaginosis (BV) is treated with metronidazole 500 mg orally twice a day for 7 days.

The nurse explains to the client that the obstetric conjugate measurement is important because of which reason? 1. This measurement determines the tilt of the pelvis. 2. This measurement determines the shape of the inlet. 3. The fetus passes under it during birth. 4. The size of this diameter determines whether the fetus can move down into the birth canal so that engagement can occur.

Answer: 4 Explanation: 4. The obstetric conjugate extends from the middle of the sacral promontory to an area approximately 1 cm below the pubic crest. The fetus passes through the obstetric conjugate, and the size of this diameter determines whether the fetus can move down into the birth canal in order for engagement to occur.

The nurse is preparing a handout on the ovarian cycle to a group of middle school girls. Which information should the nurse include? 1. The hormone human chorionic gonadotropin stimulates ovulation. 2. Irregular menstrual cycles have varying lengths of the luteal phase. 3. The ovum leaves its follicle during the follicular phase. 4. There are two phases of the ovarian cycle: luteal and follicular.

Answer: 4 Explanation: 4. The ovarian cycle has two phases: the follicular phase (days 1 to 14) and the luteal phase (days 15 to 28 in a 28-day cycle).

The nurse is explaining the nutritional differences between breast milk and formula to an expectant couple. The mother-to-be asks whether breast milk is nutritionally superior to formula. What should the nurse reply? 1. The vitamins and minerals in formula are more bioavailable to the infant. 2. There is no cholesterol in breast milk. 3. The only carbohydrate in breast milk is lactose. 4. The ratio of whey to casein proteins in breast milk changes to meet the nutritional needs of the growing infant.

Answer: 4 Explanation: 4. The ratio of whey to casein proteins in breast milk, unlike that in formula, is not static. It changes to meet the nutritional needs of the growing infant.

A client in the emergency department is diagnosed with pelvic inflammatory disease. Before discharge, the nurse will provide the client with some health teaching about which topic? 1. Endometriosis 2. Menopause 3. Ovarian hyperplasia 4. IUD for contraception

Answer: 4 Explanation: 4. The woman who uses an IUD for contraception and has multiple sexual partners needs to understand clearly the risk she faces.

The nurse in the clinic is seeing four clients. Which one is the best candidate for receiving an IUC? 1. An unmarried, 22-year-old college graduate 2. A married 24-year-old who has never been pregnant 3. An unmarried 25-year-old with a history of chlamydia 4. A married 26-year-old who has two children

Answer: 4 Explanation: 4. This client is the best candidate, as she is in a stable relationship, and has children.

The nurse educator is teaching a group of teens and 20-year-olds reproductive health care. When several of the women bring up douching, what is the best response the nurse could make? 1. "One should always douche after having intercourse." 2. "When douching, use force putting them in and get the solution up high." 3. "It is a good idea to douche before intercourse so the area is clean for the sperm." 4. "Douching is unnecessary because the lining of the vagina has numerous glands that provide natural cleansing."

Answer: 4 Explanation: 4. This is a true statement. The vagina has a natural cleansing system.

The nurse is interviewing a new client in the clinic. The client is premenopausal, but is concerned about the bone changes and osteoporosis that can occur, since she is getting close to menopause. The nurse tells the client that prevention is the primary goal. Which of the following would be a primary goal for prevention? 1. Eliminating all alcohol intake 2. Taking 500 mg of calcium each day 3. Use of sunscreen to assist with absorption of vitamin D 4. Regular weight-bearing and muscle-strengthening exercises

Answer: 4 Explanation: 4. This is correct. Regular weight bearing of the long bones is a primary goal for the prevention of osteoporosis.

A client is asking the nurse what she can do about the "falling down of her reproductive organs and urinary tract wall." The nurse will tell the client to try what exercise? 1. Lifting weights to strengthen those muscles 2. Running two miles a day 3. Running up and down stairs a few times every day 4. Performing Kegel exercises and having regular sexual activity

Answer: 4 Explanation: 4. This is the correct answer. Kegel exercises are done by tightening and relaxing the perineal muscles, and this activity as well as sexual activity will help the client's problem.

The nurse is admitting a client in labor who states that she is a naturopath. The nurse understands that this client believes which of the following? 1. An initial worsening of symptoms after treatment means the correct remedy has been used. 2. There are five elements that take form in the body. 3. Her pregnancy is a kapha condition. 4. Naturopathy is a form of medicine that utilizes the healing forces of nature.

Answer: 4 Explanation: 4. This is the correct answer. Naturopathy is more precisely defined as a healing system that combines safe and effective traditional means of preventing and treating human disease with the most current advances in modern medicine.

A 56-year-old client comes into the gynecology clinic with the complaints of constipation and a protrusion from her vagina. What does this client most likely have? 1. A cystocele 2. A prolapsed uterus 3. Polycystic ovarian syndrome 4. A rectocele

Answer: 4 Explanation: 4. This is the correct answer. The client often complains of constipation, and the anterior wall of the rectum protrudes through the vagina.

A 12-year-old girl and her mother are at the doctor's office for a routine check-up for the daughter. The mother tells the nurse that she would like the daughter to have the Gardasil vaccine that is effective against the human papilloma virus. The nurse does some teaching, and knows it has been successful when the mother makes which statement? 1. "The human papilloma virus is spread through casual contact in schools." 2. "Gardasil will protect against all types of the human papilloma virus." 3. "The human papilloma virus affects a million people in the United States." 4. "Gardasil will be given to my daughter in three doses."

Answer: 4 Explanation: 4. This is true. The vaccine is given in three doses.

The nurse is working with a client from Southeast Asia. The client tells the nurse that she should not put the baby to breast until her milk comes in and her breasts are warm, because "cold milk" (colostrum) is bad for the baby. After the nurse explains the benefits of colostrum, the client still insists that "cold milk" is bad. Which response by the nurse is best? 1. "What kind of formula would you like to use?" 2. "That idea is folklore. Colostrum is good for the baby." 3. "Now that you are here, you need to feed your baby the right way." 4. "Let's give the baby formula after you breastfeed."

Answer: 4 Explanation: 4. This response attempts to provide a compromise between acknowledging the client's desire to give formula and getting the baby to breast to get colostrum. Nurses should be aware that some immigrant mothers may have this misconception about their colostrum.

A female client who is 36 years old, weighs 200 pounds, is monogamous, and does not smoke desires birth control. The nurse understands that which contraceptive method is inappropriate for this client? 1. Intrauterine device 2. Vaginal sponge 3. Combined oral contraceptives 4. Transdermal hormonal contraception

Answer: 4 Explanation: 4. Transdermal hormonal contraception is contraindicated because of the client's obesity.

The nurse is preparing to assess the development of a family new to the clinic. The nurse understands that which of the following is the primary use of a family assessment tool? 1. Obtain a comprehensive medical history of family members. 2. Determine to which clinic the client should be referred. 3. Predict how a family will likely change with the addition of children. 4. Understand the physical, emotional, and spiritual needs of members. Answer: 4

Answer: 4 Explanation: 4. Understanding the physical, emotional, and spiritual needs of members is the main reason for using a family assessment tool.

A couple asks the nurse what is the safest method of sterilization. What should the nurse reply? 1. "Laparotomy tubal ligation." 2. "Laparoscopy tubal ligation." 3. "Minilaparotomy." 4. "Vasectomy."

Answer: 4 Explanation: 4. Vasectomy (male sterilization) is a relatively minor procedure.

The client has been diagnosed with hepatitis B. Which statement indicates to the nurse that the client needs more education? Note: Credit will be given only if all correct and no incorrect choices are selected. Select all that apply. 1. "This infection could be sexually transmitted." 2. "I might get jaundiced from this illness." 3. "An immunization exists to prevent getting hepatitis B." 4. "I might have gotten this infection from food." 5. "The incubation period is 15-50 days."

Answer: 4, 5 Explanation: 4. Hepatitis B is found in blood and body fluids, and therefore can be sexually transmitted. Hepatitis A and E are foodborne, and transmitted by fecal-oral contamination. 5. The incubation period for hepatitis B is 45-160 days.

The nurse is taking a history of a new client in the clinic. Histories tend to be lengthy, and the sexual part can be difficult for the client. The nurse should use what technique to make it easier for the client? 1. Let the client fill out a paper copy, so she does not have to talk about intimate matters. 2. Skip the sexual part until the next time the client comes into the clinic. 3. Start with the easier medical and surgical questions, and develop a feeling of trust with the client. 4. Leave the sexual part of the history for the doctor to ask about. Answer: 3

Explanation: 3. Starting with easy-to-answer questions and then going to the sexual ones helps, as client might be at ease by then.

) Which client would the nurse document as exhibiting signs and symptoms of primary dysmenorrhea? 1. 17-year-old, has never had a menstrual cycle 2. 16-year-old, had regular menses for 4 years, but has had no menses in 4 months 3. 19-year-old, regular menses for 5 years that have suddenly become painful 4. 14-year-old, irregular menses for 1 year, experiences cramping every cycle

Explanation: 4. Dysmenorrhea, or painful menstruation, occurs at, or a day before, the onset of menstruation and disappears by the end of menses. Primary dysmenorrhea is defined as cramps without underlying disease.


Kaugnay na mga set ng pag-aaral

History, Section Quiz, Ancient Greece.

View Set

CAP Ground School Aerodynamics Study Questions

View Set

Auditing - Compilation Engagements

View Set